Vous êtes sur la page 1sur 57

SPÉCIALE MP* : ORAL 2006

1. Sujets posés aux Écoles Normales Supérieures à l’oral 2006


1.1. Oral Maths Ulm.
Exercice 1.1.1. On pose Z2 l’ensemble des rationnels dont le dénominateur est impair (dans
leur écriture propre) et O3 (Q) l’ensemble des matrices orthogonales à coefficients rationnels.
Montrer que tous les coefficients d’une matrice de O3 (Q) appartiennent à Z2 .

Exercice 1.1.2. Soit (G, ×) un groupe et H un sous-groupe de G.


On considère E = {aH, a ∈ G} supposé fini de cardinal p premier et la propriété
(⋆) ∀a ∈ G \ H, a2 , ..., ap−1 ∈
/ H.

(1) On suppose (⋆), montrer que H est distingué dans G (i.e. ∀g ∈ G, ∀h ∈ H on a


ghg −1 ∈ H)
(2) On ne suppose plus (⋆), mais G est fini et p est le plus petit facteur premier du cardinal
de G. Même question.
(3) Réciproque : on suppose encore G fini et H distingué dans G, a-t’on (⋆) ?
Question bonus : montrer que si Card G = p alors G est isomorphe à Z/pZ (avec peut-être
d’autres hypothèses).

Exercice 1.1.3.
Soit E = Rn [X] muni de la norme kP k = sup |P (x)|, on pose ϕa (P ) = P (a) et αn (a) = kϕa k
x∈[−1,1]
(norme subordonnée).
Étudier αn (a) en fonction de n et a (minorer, majorer, calculer).

Exercice 1.1.4. Soit f : R → R∗+ dérivable en 0 et telle que f (0) = 1. On définit la suite
(an ) par a1 > 0 et an+1 = an f (an ).
P
(1) Nature de an ?

1.2. Oral Maths Ulm-Lyon-Cachan.


Exercice 1.2.1. Soit f une fonction continue de Rn dans R. On suppose que l’image réciproque
de tout singleton est un compact. Est-ce que f admet un maximum ou un minimum global?

1
2 SPÉCIALE MP* : ORAL 2006

Exercice 1.2.2.
(1) On se place dans Mn (R). Quel est l’espace vectoriel engendré par les matrices nilpo-
tentes ?
(2) Trouver A et B telles que AB = 0 et BA 6= 0
(3) Montrer qu’il n’existe pas de norme invariante par similitude (i.e. telle que N(A) =
N(P AP −1) pour tout P inversible)
(4) Montrer que N(A) = |Tr A| est une semi-norme (i.e. une norme telle que N(A) = 0 ;
A = 0)
(5) Trouver toutes les semi-normes invariantes par similitude.

Exercice 1.2.3. Soit (a, b, c, A, B, C) ∈ R6 , on suppose que


∀x ∈ R, |ax2 + bx + c| 6 |Ax2 + Bx + C|.
Montrer que |b2 − 4ac| 6 |B 2 − 4AC|.

Exercice 1.2.4.
(1) Soient A, B et M trois matrices de Mn (C) telles que AM = MB et Rg(M) = r.
Montrer que A et B ont r valeurs propres communes (comptées avec leur ordre de
multiplicité dans le polynôme caractéristique).
(2) Soient A et B dans Mn (C) telles que AB = BA. On note ΠA le polynôme minimal de
A, ΠB celui de B. Montrer que deg(ΠAB ) 6 deg ΠA . deg ΠB .
(3) Soient A, B des matrices de Mn (C). Que dire des polynômes caractéristiques de AB
et BA ?

Exercice 1.2.5. Soit E un C-espace vectoriel de dimension n, u ∈ L(E). On note ur


l’endomorphisme induit par u sur le R-espace vectoriel associé à E.
(1) Exprimer det ur .
(2) À la fin :
a) Est-ce que [0, 1[ est homéomorphe a ]0, 1[ ?
b) B(0, 1) est-elle homéomorphe à B(0, 1) ∪ {(1, 0)} (sur R2 ) ?

1.3. Oral Maths Lyon.


P
n−1
Exercice 1.3.1. Soit P (X) = X n − ai X i avec les ai > 0 non tous nuls.
i=0
(1) Montrer que P a une racine strictement positive.
(2) Soit ρ la plus grande racine strictement positive de P . Montrer que ρ < max {ai } + 1
(3) Soit λ une racine quelconque de P . Montrer que |λ| < ρ.
(4) Montrer que ρ est de multiplicité 1.
(5) Montrer que ρ est la seule racine réelle strictement positive.
SPÉCIALE MP* : ORAL 2006 3

Exercice 1.3.2.
(1) Existe-t-il des matrices de GLn (C) distinctes de l’identité semblables à leur carré ?
(2) Même question dans GLn (R).
(3) Trouver toutes les matrices de GL2 (C) semblables à leur carré.
(4) Même question dans GL2 (R).

Exercice 1.3.3. Soit A un anneau commutatif, on définit


n
X
n
Sn (A) = {x ∈ A | ∃(a1 , . . . , an ) ∈ A , x = a2i }.
i=1

(1) S2 est-il stable par la multiplication ?


(2) S3 est-il stable ? (
n∈ / S3 (Z)
(3) Montrer que, si n ≡ 7[8] alors
n∈ / S3 (Q)
(4) En effaçant le tableau : est-ce que [0, 1] est homéomorphe à ]0, 1[ ?

1.4. Oral Maths Cachan.


Exercice 1.4.1.
(1) a) Soit H ∈ Mn (R) symétrique. Calculer det(I + tH) pour t ∈ R.
b) Soit A et B réelles symétriques définies positives, A 6= B et 0 < α < 1.
Montrer que det (αA + (1 − α) B) > (det A)α (det B)1−α .
c) Que dire de {M ∈ Mn (R) symétrique définie positive | det M > 1} ?
(2) Soit j ∈ N et h ∈ R tel que jh = 1, (un ) ∈ RN avec u0 = 0.
P
j P uk+1 −uk 2
j−1
Montrer que u2k 6 h
.
k=0 k=0

Exercice 1.4.2.
(1) Soit ϕ ∈ C(R
Z+ , R), montrer que, s’il existe C ∈ R tel que, pour tout t ∈ R+ , on a
t
tϕ(t) 6 C + ϕ(s) ds alors ϕ est bornée.
0
(2) Montrer que si f vérifie l’équation différentielle f ′′ + qf =? alors f est bornée.
 
2 1 0 1
1 2 1 0
(3) Soit M =  
0 1 . . . 1, montrer que M est positive non définie.
1 0 1 2

Exercice 1.4.3.
P 1
Soit a > 0, on pose f (t) = 2 2
.
n∈Z a + (t − 2nπ)
(1) Définition, régularité, parité, périodicité de f .
(2) Calcul des coefficients de Fourier de f .
(3) Si ap désigne le coefficient de Fourier de f , calculer ϕ(a) = aπap .
4 SPÉCIALE MP* : ORAL 2006

Exercice 1.4.4.
∂f ∂f
(1) Soit f : R2 → R, différentiable et vérifiant f (0, 0) = 0 ainsi que
> | |. Montrer
∂y ∂x
que ∀x ∈ R, ∃!y ∈ R, f (x, y) = 0. Si on définit g : x →
7 y, montrer que g est alors
continue et dérivable.
(2) Connaissez-vous des démonstrations du théorème des fonctions implicites? (question
”culturelle”) Comment le prouver avec le même genre de démonstration qu’en (1)?

2. Spéciales MP* : sujets posés à l’école Polytechnique à l’oral 2006


2.1. Mr Grigis.
Exercice 2.1.1.
(1) Soit P = a0 + a1 z + . . . + an z n un polynôme à coefficients entiers tels que 0√6 ai 6 9 et
1
an > 1. Montrer que si z est une racine de P alors P R(z) 6 0 ou |z| 6 2 (1 + 37).
(2) P
Soit une matrice carrée de taille n telle que ∀j, i mij = 1. On prend x un vep tel que
i xi 6= 0. Trouver la valeur propre associée. (+ question donnée à la fin : si la somme
sur chaque ligne vaut 1, que peut-on dire?)

Exercice 2.1.2.
E = C ∞ (R, C). On pose, pour toute f dans E, fa (x) = f (x + a) et Ff l’espace engendré par
les fa . On note Φ = {f ∈ E, dim(Ff ) < +∞} .
Déterminer Φ. (T’es seul, t’es seul).

Z +∞
x
Exercice 2.1.3. Soit f (x) = eit dt pour x > 1.
0
(1) Étudier la convergence.
(2) Donner un équivalent en +∞.

Exercice 2.1.4. Soit g(x) = | sin x|.


1 P cos 2nx
4 +∞
(1) Montrer que g(x) = − .
π π n=1 4n2 − 1
(2) Montrer que l’équation (E) : y ′′ + y = g(x) admet une unique solution C 2 , C 3 par
morceaux et π-périodique.

Exercice 2.1.5. Soit n ∈ N, on pose pour k ∈ [[1, n]] : nk le reste de la division euclidienne
de n par k et pn la probabilité pour que nk > k2 .
(1) Calculer pn .
(2) Calculer lim pn .
n→+∞
(3) Et pour finir : nature de la suite un = n! sin(2πn!e).
SPÉCIALE MP* : ORAL 2006 5

2.2. Mr Henry.
Exercice 2.2.1.
(1) Soit u ∈ L(Kn ) tel que u(e1 ) = 0 et u(ek ) = ek−1 pour k ∈ [[2, n]]. Trouver les sous-
espaces stables par u.
(2) Soif f : R −→ R lipschitzienne. Montrer que f s’écrit comme la différence de deux
fonctions monotones.
(3) Soit f : [0, 1[−→ R uniformément continue. Montrer que f est bornée.
(4) Est-ce que la fonction ln peut s’écrire localement comme un polynôme ?

Exercice 2.2.2.
R π/2
(1) Soit wn = 0 sinn x dx. Étude.
(2) a) Soit P ∈ Q[X]. P irréductible dans Q. Montrer que P n’admet pas de racine
double.
b) Soit P ∈ Q[X] de degré 5 qui admet une racine double. Montrer que P admet une
racine rationnelle. P P
(3) Soit (xn ) positive et (an ) ց 0 telles que. an xn converge. Étude de an nk=1 xk

2.3. Mr Langevin.
Exercice 2.3.1.
Q
n
(1) Soit A ∈ Mn (C). On note C1 , ..., Cn ses colonnes. Montrer que | det A| 6 ||Ci ||.
i=1
(2) Soit un définie par u0 , u1, u2 et un+3 = aun+2 + bun+1 + cun . On suppose a + b + c = 1
et un convergente, trouver sa limite. (On pourra traiter le cas un+2 = aun+1 + bun avec
a + b = 1 et essayer de généraliser).

Exercice 2.3.2. Que dire de f : (A, B, C) ∈ E 3 7→ (α, β, γ) où E est le plan affine euclidien
−→ −→ −→ −−→ −→ −−→
et α = AB.AC, β = BA.BC, γ = CA.CB.

Exercice 2.3.3. Soient A, B, C, D 4 points du plan tels que


0 AB 2 AC 2 AD 2
2
BA 0 BC 2 BD 2
=0
CA2 CB 2 0 CD2
DA2 DB 2 DC 2 0
Montrer que A, B, C, D sont cocycliques!

Exercice 2.3.4.
(1) Déterminer 3 nombres α, β, γ tels que f : C → C soit constante où
f (z) = α(z − a)2 + β(z − b)2 + γ(z − c)2 .
Si A, B, C sont trois points alignés du plan, −

u un vecteur unitaire de la droite passant
par ces trois points alors montrer que
−−→ −→ −→
MA2 BC + MB 2 CA + MC 2 AB = ±AB.BC.CA.− →
u.
6 SPÉCIALE MP* : ORAL 2006

(2) Soit A ∈ Mn (C), on suppose qu’il existe p1 6= p2 tels que Ap1 = Ap2 . Montrer qu’il
existe p tel que Ap soit diagonalisable.

2.4. Mr Rosso.

Exercice 2.4.1. On prend une matrice P symétrique positive. Montrer que :


Y
det P 6 pii .
i

Exercice 2.4.2.
X(X−1)···(X−j+1)
Soit ∀j ∈ N, Pj (X) = j!

(1) Soit P ∈ Rk [X]/ ∃n ∈ N, P (n, n + 1, · · · , n + k) ⊂ Z. Montrer que l’on peut écrire


P
P (X) = k0 aj Pj (X) où les aj sont des entiers.
(2) Soit R une fraction rationnelle telle que R(Z) ⊂ Z. Montrer que R est un polynôme.

Exercice 2.4.3.
Soient A = (aij ) et B = (bij ) deux matrices symétriques réelles d’ordre n, on définit la matrice
C de coefficients cij = aij bij .
Montrer que
(1) Si A > 0 et B > 0 alors C > 0.
(2) Si A > 0 et B > 0 alors C > 0.
(3) A > 0 ⇔ (∀B > 0, C > 0).

Exercice 2.4.4.
Soit f ∈ C 3 (Rn , R) telle que f (0) = 0 et f ′ (0) = 0.
Montrer qu’il existe h ∈ C 1 (Rn , Sn ) telle que ∀x ∈ Rn , f (x) = xT h(x)x (Sn désigne l’ensemble
des matrices symétriques réelles).

Exercice 2.4.5.
P
n
(1) Soit σ ∈ R∗+ , (ak ) ∈ RN telle que Sn = ak k −σ converge.
k=1
−σ
P
n
Montrer que n ak → 0.
k=1
(2) Soit J = [[1, n]], z1 , . . . , zn n complexes.
P 1P
Montrer qu’il existe I ⊂ J tel que zi > |zi |.
i∈I 6 i∈J
SPÉCIALE MP* : ORAL 2006 7

2.5. Divers.
Exercice 2.5.1. Soit A une matrice symétrique définie positive, on pose
K(x) = (Ax|x)(A−1 x|x)
et on se propose de déterminer min K(x) et max K(x).
kxk=1 kxk=1
(1) Montrer que si K passe par un extremum en x ∈ S(0, 1) alors il existe λ ∈ R tel que
(Ax|x)A−1 x + (A−1 x|x)Ax = λx.
(2) En déduire que x appartient à la somme d’au plus 2 sous-espaces propres de A.
(3) Conclure.

Exercice 2.5.2.
a(1 + a2 )
(1) Nature de la suite définie par u0 ∈ R et la relation de récurrence un+1 = ,
1 + u2n
a > 0.
(2) Soit f ∈ L(E) où E est un espace vectoriel de dimension finie. Montrer l’équivalence
Ker f = Im f ⇔ (f 2 = 0) et (∃h ∈ L(E) | h ◦ f + f ◦ h = IdE .

3. Spéciales MP* : sujets posés aux Écoles des Mines à l’oral 2006

Exercice 3.1.1.
(1) Considérons l’équation différentielle suivante :
x2
x2 y ′′ + xy ′ − y =
1 − x2
Chercher les solutions D.S.E. (question posée à l’oral : expliciter cette solution)
 
1 ... 1
(2) Posons A =  ... . . . 0. Déterminer son spectre et ses sous-espaces propres.
1 0 1  
p q r
(3) Pour trois réels p, q, r, on pose M = r p q . Montrer que M est une matrice de
q r p
rotation ssi r, p, q sont racines du polynôme X 3 + bX 2 + c où b et c sont à déterminer.
(4) Soit (an ) une suite de réels croissante qui tend vers l’infini.
Z +∞ X +∞ X+∞
n −an x (−1)n
Montrer que (−1) e dx = .
0 0 0
an

Exercice 3.1.2.
(1) (10 min de préparation) Soit Pn (X) = X n + X n−1 + ... + X 2 + X − 1 pour n ∈ N.
a) Montrer qu’il existe un unique an > 0 tel que Pn (an ) = 0
b) Trouver la limite l de an .
c) Donner un équivalent de an − l.
(2) (Sans préparation) Montrer que la famille (fα )α∈R est libre, où fα (t) = t eαt .
8 SPÉCIALE MP* : ORAL 2006

Exercice 3.1.3.
(1) Pour n ∈ N, on considère
Z 1
xn (1 − x)
In = dx.
0 1 − xn
P
Étudier la suite (In )n∈N , puis la série In .
n
(2) On pose E = R , où n ∈ N. Soit u ∈ L(E) un projecteur. On considère
Φ : L(E) −→ L(E)
f 7−→ f ◦ u + u ◦ f
Φ est-il diagonalisable ?

Exercice 3.1.4.
(1) Soit f un endomorphisme de E tel que f 2 + f − 2 IdE = 0, montrer que
E = Ker(f + 2 IdE ) ⊕ Ker(f − IdE ).
P
+∞ 1
(2) Soit g(x) = x
.
n=1 n
Chercher l’ensemble de définition de g, étudier sa dérivabilité, donner l’expression de sa
dérivée et étudier
 le comportement
 de g aux bornes de son ensemble de définition.
r p q
(3) Soit M = p q r . Montrer que M est une matrice de rotation ssi r, p, q sont
q r p
racines du polynôme X 3 + bX 2 + c où b et c sont à déterminer.

Exercice 3.1.5.
(1) Soit P ∈ R[X], unitaire.
a) Montrer que : P scindé sur R ssi ∀z ∈ C, |P (z)| > | Im(z)|deg P .
b) Soient E un R-e.v., (Um ) une suite d’endomorphismes diagonalisables qui converge
vers U. U est-il diagonalisable ? Trigonalisable ?
(2) Soit E = C([0, 1]), A = {f ∈ E | f (0) = 0}.
Montrer que A est ou bien dense dans E ou bien fermé

Exercice 3.1.6.
(1) Soit t ∈]0, 1[, f (x,
Z y) = t(2x+1)y ,Zmontrer que f ∈ L1 ([1, +∞[2).
+∞ 3y +∞ 2x+1
t t
En déduire que dy = dx.
1 2y 1 2x + 1
(2) Soit E un espace euclidien, p un projecteur.
a) Montrer que p est orthogonal ssi ∀x ∈ E, kp(x)k 6 kxk.
b) En déduire que l’ensemble K des projecteurs orthogonaux est compact.
(3) Étudier la courbe définie en polaires par r = cos(2θ)...
SPÉCIALE MP* : ORAL 2006 9

Exercice 3.1.7. 

x = u cos v
(1) Soit S la surface d’équations paramétriques : y = u sin v où a ∈ R, ϕ ∈ C 1 (R, R).

z = au + ϕ(v)

a) Trouver l’équation du plan tangent en chaque point.


b) Montrer que, à v fixé, il existe une droite qui appartient à tous les plans lorsque u
décrit R. P P un P P R un dx
(2) Soit (un ) ∈ (R+ )N , montrer que un , 1+un
, ln(1 + u n ) et 0 1+x2
sont toutes
de même nature.

Exercice 3.1.8.
(1) Résoudre dans (Z/36Z)2 : (
5.5̇x − y = 11 ˙
3̇x + 5̇y = 1̇
(10 min de préparation).
(2) Soit f ∈ C 2 (R+ , R+ ), bornée. On suppose qu’il existe α ∈ R+ tel que f ′′ > α2 f . Montrer
que f et f ′ tendent vers 0 en +∞.
(3) Donner l’équation cartésienne de la surface de révolution Σ d’axe Oz engendrée par la
courbe (
x+z =1
C 2 2 2
.
(x − 1) + y + z = 1
(4) Donner l’équation cartésienne d’un hyperboloı̈de (à 2 mn de la fin).

Exercice 3.1.9.
(1)
a) Soit E = C ∞ (R, C) et D : f 7→ f ′ . Existe-t-il ∆ ∈ L(E) tel que ∆2 = D ?
b) Si E = Vect(cos,
Z +∞ sin), existe-t-il ∆ tel que ∆2 = D ?
sin xt
(2) Soit f (t) = dx.
0 x(1 + x2 )
a) Montrer que f est de classe C 1 sur [0, +∞[.
b) Montrer que f est deux fois dérivable sur ]0, +∞[ et que f satisfait l’équation
π
différentielle y − y ′′ = , en déduire l’expression de f sur [0, +∞[.
2
c) Quelle est l’expression de f sur R ?

Exercice 3.1.10.
(1) Soit ϕ : Rn [X] → R telle que ϕ(P ) = P (a) où a est un réel fixé.
a) Montrer que ϕ continue.
b) Calculer la norme subordonnée de ϕ pour la norme 1 et la norme infinie.
10 SPÉCIALE MP* : ORAL 2006

(2) Exercice assez simple sur les séries de fonctions (étant donné un terme général, domaine
de définition, convergence, limites).
(3) On considère la courbe intersection des deux surfaces :
x2 + y 2 − 4z = 9 et x + y − 2z = 0.
Équation de la surface de révolution autour de Oz.
(4) Soit A une matrice réelle telle que A4 + 5A2 + 4In = 0.
Montrer que Tr A = 0.

4. Spéciales MP* : sujets posés aux Écoles Centrales à l’oral 2006

4.1. Math 1.
Exercice 4.1.1.
(1) (Avec préparation) : Soit A ∈ On (R) de coefficients aij.
P √
a) Montrer que |aij | 6 n n.
i,j
 
1
P
b) En considérant v =  ... , montrer que aij 6 n.
i,j
1
c) Peut-on avoir égalité à la fois dans a. et b. ?
d) Quelle transformation géométrique admet une matrice symétrique et orthogonale ?
(2) (Sans préparation) : (Z/4Z)2 est-il cyclique ?
(3) Soit (f1 , f2 ) ∈ (Rn⋆ )2 .
Montrer l’équivalence : (f1 , f2 ) est libre ⇔ dim Ker f1 ∩ Ker f2 = n − 2.

Exercice 4.1.2.
(1) Soit n ∈ N, E = Rn , u et v deux endomorphismes de E qui commutent.
Soit χu et χv leurs polynômes caractéristiques.

a) On suppose que χu est scindé et que χu est premier avec χu , montrer que v est
diagonalisable.
b) Est-ce une condition nécessaire ?

c) Contre exemple (v non-diagonalisable) avec χu non premier avec χu .
d) Contre exemple (v non-diagonalisable) avec χu non scindé.
e) Reprendre les questions précédentes avec E = Cn .
(2) Quels sont les sous-groupes de (Z/nZ, +) ?

Exercice 4.1.3. Soit f ∈ L(E) telle que f admette un polynôme annulateur P .


(1) Montrer que f admet un polynôme minimal.
(2) Montrer que si f est inversible alors f −1 est un polynôme de f .
(3) Donner un endomorphisme qui n’admet pas de polynôme annulateur.
(4) Soit M ∈ Mn (R), est-ce que toutes les matrices M admettant X 6 + X 4 + X 2 + 1 comme
polynôme minimal sont semblables entre-elles ?
Discuter selon les valeurs de n (au passage, il a demandé de prouver que si 2 matrices
de Mn (R) sont semblables dans C alors elles sont semblables dans R).
SPÉCIALE MP* : ORAL 2006 11

Exercice 4.1.4.
(1) Soit M ∈ Mn (C).
On note C(M) = {P (M), P ∈ C[X]} et IM = {A ∈ C(M) | A2 = In }.
Montrer que la dimension de C(M) est égale au degré du polynôme minimal de M.
Déterminer le cardinal de IM dans le cas particulier où M est diagonalisable.
(2) Déterminer de même Card IM lorsque M est nilpotente.
(3) Étudier le cas général.

Exercice 4.1.5. On dit que x est u-générateur de F (sev de E) ssi F est le plus petit sev de
E qui vérifie : x ∈ F et u(F ) ⊂ F .
 
2 1 2
(1) Dans R , quels sont les u-générateurs, avec u = .
1 1
(2) Montrer que : x est un u-générateur de F ssi F = {f (x), f ∈ R[u]}.
(3) Soit up = u − p. Id, montrer que les u-générateurs sont les up -générateurs.
(4) Soit C(u) = {v | u ◦ v = v ◦ u}. Montrer que R[u] ⊂ C(u) et que C(u) est un sev.
(5) Si Pu (X) est scindé à racines simples, alors montrer que R[u] = C(u).
(6) Questions complémentaires :
a) Donner une matrice non diagonalisable de M3 (C).
b) Donner le reste de la division euclidienne de X n par (X 3 − 1).
c) Combien y a-t-il d’automorphismes de Z/3Z dans Z/3Z ?

Exercice 4.1.6. Soit A = (aij ) ∈ Mn (R), on suppose que aij ∈ {0, 1}, aii = 0 et, si i 6= j,
aij + aji = 1.
(1) Soit H l’hyperplan d’équation x1 + · · · + xn = 0, chercher Ker A ∩ H.
(2) En déduire que Rg(A) > n − 1.
(3) Quels sont les entiers n > 2 tels que toutes les matrices A soient de rang n − 1 ?
Questions annexes :
(1) Quelles sont les vap de J ?
(2) Montrer que la matrice A de la fin est bien de rang n.
(3) Quel est le polynôme caractéristique de J, quel est son polynôme minimal ?
(4) Trouver une matrice de polynôme minimal X 2 + 1.

Exercice 4.1.7.  
1 −1 −1
(1) On considère A = −1 1 −1.
−1 −1 1
a) Chercher l’idéal annulateur de A, en déduire R[A].
b) Trouver (an ) et (bn ) tels que ∀n ∈ N, An = an I3 + bn A.
c) Soit CR (A) le commutant de A dans M3 (R), déterminer dim CR (A).
d) Déterminer {B ∈ M3 (R) | B 2 = A}.
(2) Soit A ∈ Mn (C), montrer que dim CC (A) > n.
12 SPÉCIALE MP* : ORAL 2006

Exercice 4.1.8.
(1) On considère l’équation X 2 = A dans Mn (R)
a) n = 2, trouver A telle qu’on ait
• une infinité de solutions,
• aucune solution
• un nombre finide solutions.
  
0 1 0 2 0 0
b) Résoudre pour A = 0 0 1 et A = 1 2 0.
0 0 0 0 1 3
c) Que dire si χA (pol caractéristique) scindé à racines simples ?
d) Que dire si χA scindé à racines positives ?
e) Que dire si χA scindé mais n’a pas toutes ses racines positives ?
f) Résoudre dans Mn (C) X 2 = A pour A diagonalisable.
g) Résoudre Xn2 = In dans Mn (R).
(2) Montrer que la famille (sin xn )n>0 est libre...
(3) Calcul de cos 2π
5
.

Exercice 4.1.9.
(1) a) Montrer qu’il existe un unique polynôme Tn tel que ∀x ∈ R, Tn (cos x) = cos nx.
b) Établir une relation de récurrence entre Tn+2 , Tn+1 et Tn .
c) Chercher le degré, la parité et le terme dominant de Tn .
Z 1
P (x)Q(x)
d) Montrer que ϕ(P, Q) = √ dx est un produit scalaire sur R[X].
−1 1 − x2
e) Calculer ϕ(Tp , Tq ), en déduire une base orthonormale de R[X].
f) Matrice de la réflexion orthogonale par rapport à Rn−1 [X] dans cette base.
(2) Décrire l’ensemble des matrices de M2 (R) admettant X 2 +1 comme polynôme minimal.

4.2. Math 2.

Exercice 4.2.1.
(1) (Avec préparation) : On considère une suite an décroissante de réels strictement positifs,
et un (t) = an tn (1P
− t) pour n ∈ N et t ∈ [0, 1].
a) Montrer que un converge simplement sur [0, 1].
b) Trouver une CNS P sur an pour avoir convergence normale.
c) Montrer que un converge uniformément ⇔P an −→ 0.
d) On suppose an 9 0. étudier la continuité de un .
P C.U.
(2) (A 5min de la fin) : On considère fn −→ f et fn (t) −→ αn .
[a;+∞[ t−→+∞
Que dire de f pour t −→ +∞ ?

Exercice 4.2.2.
(1) a) Soit un et vn deux
Psuites à termes réels positifs.
On suppose que un vn converge simplement.
SPÉCIALE MP* : ORAL 2006 13

On définit la suite :
 
1 −v1
 .. 
 u1 1 . (0) 
 .. .. .. 
∆n = det 
 . . . 

 .. .. 
(0) . . −vn 
un 1
On pose ∆0 = 1. Montrer que (∆n ) converge.
n
Y
(On pourra considérer la suite wn = (1 + uk vk ).)
k=1
b) Réciproque
(2) Calculer
Z √
n  n
t √
lim 1− √ et n
n 0 n

Exercice 4.2.3.
P
+∞ 1 1
(1) Montrer que =
n=1 n(n + 1)(. . .)(n + q) q.q!
1 1
(considérer − ).
n(n + 1)(. . .)(n + q − 1) (n + 1)(. . .)(n + q)
(2) Montrer que
q +∞
π2 X 1 X 1
= 2
+ q! 2
.
6 n=1
n n=1
n (n + 1)(. . .)(n + q)
(3) Donner une méthode pour avoir une valeur approchée de π à 10−100 .

Exercice 4.2.4.
Z b
1
(1) Soit f ∈ C ([a, b], R). Calculer lim f (t) sin nt dt.
n→+∞ a
Z π/2
cos x
(2) Calculer lim sin(2nx) dx.
n→+∞ 0
Z 2π sin x
(3) Calculer lim ln(2 sin(x/2)) cos nx dx et chercher son équivalent en +∞.
n→+∞ 0
1 cos(x/2) 1
Indication pour le 2 : considérer θ(x) = − .
2 sin(x/2) x

Exercice 4.2.5.
P
+∞
2 x2
(1) Soit f (x) = e−n .
n=0
a) Domaine de définition de f .
b) Trouver lim xf (x).
x→0
c) Trouver lim f (x).
x→+∞ P
(2) Nature de la série un où un = cos[πn2 ln(1 − 1/n)].
14 SPÉCIALE MP* : ORAL 2006

Exercice 4.2.6.
1 1
(1) On considère S = + + ···.
1.2.3.4 5.6.7.8
a) Justifier la convergence. Peut-on calculer S avec Maple ?
A
b) Équivalent du reste sous forme d’une intégrale puis sous la forme α .
n
x
(2) Soit un (x) = 2 , n > 1, x > 0.
n + x2 P
a) Étudier la convergence simple, normale sur tout segment de un .
P
+∞ x
b) Comparer un (x) avec une intégrale faisant intervenir fx (t) = 2 , x étant
n=1 t + x2
fixé.
c) Conclure pour la convergence uniforme sur R+ .

Exercice 4.2.7.
(1) Exercice ultra simple sur les séries entières
(2) Soit u0 > 0, u1 > 0 et un+1 = 1+uunnun−1 .
Équivalent de un .

Z 1
xn
Exercice 4.2.8. Soit un = n−1 )2
dx.
0 (1 + x + · · · + x
(1) Convergence de la suite (un ).
(2) Écrire un comme somme d’une série, en déduire un équivalent (?).
SPÉCIALE MP* : ORAL 2006 1

Solution 1.1.1 (R. Portalez) Note : ?


Examinateur : D. HARARI, petit, un peu grassouillet, gentil, avare en paroles mais ne laisse
pas t’es seul t’es seul non plus, ce qu’il aurait pu faire facilement vu l’exo. C’est le ”Daf en
plus vieux” de Carla. Il est assez malsain et fait des blagues pendant l’oral, il m’a parlé de la
coupe du monde... Donne des indications et sur un exo comme ça, il vaut mieux! Avant, il
était chargé de cours d’algèbre à ULM.
On raisonne par l’absurde en considérant un vecteur colonne de la matrice. On sait qu’il est
p2 p221 p231
unitaire et donc q11 2 + q2 + q2 = 1. On suppose donc que q11 est pair. On distingue alors
11 21 31
plusieurs cas.
Supposons que q21 et q31 soient impairs. On met tout au même dénominateur et on obtient :
p211 q21
2 2
q31 + p221 q11
2 2
q31 + p231 q112 2
q22 = q11 2 2 2
q21 q31 . On a clairement une absurdité puisque p11 est
impair.
Il existe alors un deuxième dénominateur pair. On suppose que c’est le deuxième, histoire de
pas s’embrouiller.
Là, il faut penser à la valuation 2−adique de q11 et q21 car le raisonnement précédent ne marche
plus. On écrit donc q11 = 2k1 q11 ′
et de même pour q21 . On met alors tout au même dénominateur
pour trouver la relation sordide :
22k2 p211 q21
′2 2
q31 + 22k1 p221 q11
′2 2
q31 + 22k1 +2k2 p231 q11
′2 ′2
q21 = 22k1 +2k2 q11′2 ′2 2
q21 q31 . On regroupe tout ça par
puissance de 2 décroissantes, à savoir :
22k1 +2k2 (p231 q11
′2 ′2
q21 −q11′2 ′2 2
q21 q31 )+22k2 p211 q21
′2 2
q31 +22k1 p221 q11 ′2 2
q31 = 0. On utilise alors que la valuation
2−adique de la somme de deux nombre vaut le min des deux valuations 2−adiques des deux
termes de la somme, sauf dans le cas où celles-ci étaient égales. Comme ici, on a la somme de
deux nombres de valuation 2−adique respectives 2k1 et 2k2 dont la somme a pour valuation
2−adique 2k1 + 2k2 au moins. Comme k2 et k1 sont supérieurs à 1 (par hypothèse), ils sont
égaux. On peut alors simplifier par 22k et on aboutit à la même absurdité que dans la premier
cas.
Les trois dénominateurs sont donc pairs, on note k1 , k2 , k3 leurs valuations 2−adiques respec-
tives. On fait le même type de raisonnement que ci-dessus et arrive à une absurdité, ce qui
prouve le résultat.
pi
Remarque : On peut faire autrement. Soient des fractions rationnelles irréductibles telles
qi
que
p21 p2n
+ · · · + = 1.
q12 qn2
On pose qi = 2ai bi où bi est un nombre impair, a = max(ai ) et E = {i ∈ [[1, n]] | ai = a}. Pour
simplifier la démonstration qui suit, on suppose que E = [[1, k]] après renumérotation.
Montrons que soit E = [[1, n]] soit 4| Card E :
• Si a = 0 alors E = [[1, n]].
p2 p2
• Si a > 0 on multiplie la relation 21 + · · · + n2 = 1 par 22a b21 . . . b2n . Dans Z/4Z, les seuls
q1 qn
carrés sont 0 et 1 donc, tout nombre impair élevé au carré est congru à 1 modulo 4. On
obtient alors
2
p21 b22 . . . b2n + · · · + p2k b21 . . . bbk . . . b2n +4.K = 22a b21 . . . b2n
| {z } | {z }
≡1[4] ≡1[4]

où K est un entier correspondant aux autres termes. Dans Z/4Z on aura donc k ≡ 0[4]
i.e. 4| Card E.
Si n = 3 alors on en déduit que E = [[1, 3]] i.e. tous les coefficients d’une matrice de O3 (Q)
appartiennent à Z2 .
Questions posées à la fin : De quelle structure algébrique peut-on munir Z 2 ? (c’est un sous
2 SPÉCIALE MP* : ORAL 2006

anneau unitaire de Q.), comment appelle-t-on la plus grande puissance de 2 qui divise un entier?
(la valuation 2−adique). (je n’en avais pas parlé pendant l’oral).

Solution 1.1.2 (Th. Pradeau) Note : 13


Examinateur : Sympa, avec une voix de fausset, assez rigoureux (il m’a repris car j’ai dit qu’un
élément de Z/nZ l’engendre ssi il est premier avec n, en fait il faut parler du représentant dans
[[0, n − 1]] de la classe d’équivalence, et encore c’est pas vrai pour 0...)
(1) Si G est commutatif c’est évident (mais ça n’aura pas d’importance). On considère
h ∈ H et g ∈ G \ H (si g ∈ H c’est immédiat), on veut montrer que ghg −1 ∈ H. On a
E = {H, gH, . . . , g p−1H}. En effet, on a bien p éléments et pour 1 < j < k < p − 1 on
a g k H 6= g j H car sinon ∃h0 ∈ H | g k = g j h0 i.e. g k−j = h0 ∈ H or k − j ∈ [|1, p − 1|]
et d’après (⋆) on a g 2 , ..., g p−1 ∈
/ H.
−1 k
Par l’absurde si ghg ∈ / H, comme (ghg −1) = ghk g −1 (récurrence immédiate), on a
aussi E = {H, ghg −1H, ..., ghp−1g −1H}. Donc ∃k ∈ [|0, p − 1|] | gH = ghk g −1 H (ce
sont deux éléments de E) et ∃h0 ∈ H | ghk g −1 = gh0 soit g −1 = h−k h0 ∈ H, soit g ∈ H,
contradiction.
(2) En fait on va montrer qu’on a (⋆). On note n le cardinal de G. Si n est pair, p = 2 et
on a (⋆) immédiatement. On suppose donc n impair.
Soit a ∈ G \ H, supposons par l’absurde que ∃k ∈ [|1, p − 1|] |ak ∈ H. On prend k
minimal (k > 2). On fait la division euclidienne : n = kq + r avec r ∈ [|0, k − 1|]. Alors
an = e = akq ar en notant e le neutre du groupe. Donc ar = a−kq ∈ H car ak ∈ H.
Comme on a pris k minimal et que r < k on a nécessairement r = 0. Donc k divise n et
comme k < p qui est le plus petit facteur premier de n, on a k = 1. Contradiction car
a∈ / H.
On a donc (⋆) et d’après (1) on a le résultat.
Z
(3) Alors d’abord l’arnaque pour montrer que si Card G = p alors G est isomorphe à pZ .
k
J’ai considéré ϕa de Z dans G tel que ϕa (k) = a . On sait que si Ker ϕa = pZ alors
Z
Im ϕa ≃ pZ . Le but c’est donc de montrer que ϕa est surjective i.e que G est monogène,
il m’a arnaqué que c’est vrai mais ça me semble louche donc il manque peut-être des
hypothèses.
Ici p est juste le cardinal de E (et plus le plus petit facteur premierde n). S’il n’est pas
Z
premier
 le résultat
 est faux : on se ramène à nZ et on prend H = 0 . Alors p = n et
E = 0 ... p − 1 , p n’est pas premier donc il y a un élément a 6= 0 d’ordre k ∈ [|2, p − 1|].
En effet s’ils sont tous d’ordre p ils engendrent tous le groupe et donc p est premier avec chacun
d’eux i.e p est premier. On a ainsi a ∈ / H, mais ak = 0 ∈ H donc (⋆) est fausse.
Si p est premier, on définit le groupe (E, ⊗) avec (aH) ⊗ (bH) = abH. Ceci n’est possible
que si H est distingué dans G (on l’a admis par manque de temps). Avec cette loi on a donc
E = {H, aH, ..., ap−1 H} d’où (⋆) (car si ak ∈ H alors ak H = H).

Solution 1.1.3 (V. Laviron) Note : ?


Examinateur : ?

Solution 1.1.4 (P. Néron) Note : ?


Examinateur : ?
P
(1) Si (an ) ne tend pas vers 0 alors an diverge.
On suppose alors que lim an = 0.
n→+∞
SPÉCIALE MP* : ORAL 2006 3

Par une récurrence immédiate, on a


n−1
Y
an = a1 f (ak )
k=1
n−1
Y
donc an → 0 ⇒ f (ak ) → 0. Comme toutes ces quantités sont strictement positives,
k=1
on peut prendre le logarithme :
n−1
X
ln f (ak ) → −∞.
k=1

Or f (ak ) = 1 + f (0)ak + o(ak ) (dérivabilité en 0) donc
ln f (ak ) = ln(1 + f ′ (0)ak + o(ak )) ∼ f ′ (0)ak .
P ′ P
On en déduit (comparaison des séries divergentes) que f (0)ak diverge donc ak
diverge.
(2) Suite au prochain numéro...

Solution 1.2.1 (R. Portalez) Note : ?


Si n = 1 le résultat est faux en prenant pour f l’identité...
Pour n > 2 le résultat est vrai. Pour le montrer on raisonne par l’absurde et on suppose donc que
f n’a ni minimum ni maximum global. On pose r1 = 1. Sur B̄(0, r1 ) qui est compacte, f admet
un max et un min qui sont respectivement atteints en x1 , x′1 . Comme ces points ne réalisent pas
de max ou de min globaux, on sait qu’il existe y1 , y1′ tels que ||y1 ||, ||y1′ || > r1 et f (y1) > f (x1 )
et f (y1′ ) < f (x′1 ). On pose alors r2 = max(r1 + 1, ||y1||, ||y1′ ||). On pose x2 , x′2 les points où
f atteint respectivement son max et son min sur la boule fermée de centre 0 et de rayon r2 .
Par récurrence on construit trois suites rn , xn , x′n telles que rn → ∞, rn−1 < ||xn ||, ||x′n || 6 rn ,
f (xn ) croı̂t strictement, f (x′n ) décroı̂t strictement. On voit alors où est le problème. En effet,
f (x1 )+f (x′1 )
en supposant f (x1 ) > f (x′1 ) (quitte à augmenter r1 ...), on prend x0 = 2
. On a alors

n > 1, f (xn ) < x0 < f (xn ). Comme ensuite la couronne rn−1 < ||x|| 6 rn est connexe par arcs,
on peut rejoindre xn et x′n par un arc continu γn tel que γn (0) = xn , γn (1) = x′n . En appliquant
le T.V.I à f ◦ γn , on trouve un tn tel que f (γn (tn )) = x0 . Mais pas de bol, la suite γn (tn ) diverge
ce qui est contradictoire avec l’hypothèse...
Commentaire : J’ai fait cet exo sans aide en à peu près 35 minutes et il m’a alors collé un truc
de matrices hermitiennes sur lequel je ne connaissais rien... Comme quoi le hors programme ça
peut servir (ou pas).
Remarque : il y a moins pénible comme démonstration !
On peut supposer f (0) = 0 (par translation). K = f ( − 1)(0) est un compact donc il existe
R > 0 tel que K ⊂ B(0, R).
A = Rn \ B(0, R) est connexe par arcs et f ne s’annule pas sur A donc f est de signe constant
sur A (on peut supposer f > 0 sur A).
Par compacité, il existe x0 ∈ B(0, R) tel que f atteint son minimum en x0 sur cette boule. On
a alors f (x0 ) 6 f (0) = 0 et f atteint son minimum global en x0 .

Solution 1.2.2 (Th. Pradeau) Note : 14


Examinateur : celui de la salle Lebesgue, super sympa. C’était mon dernier jour d’oral (et
encore de l’algèbre !) et il m’a demandé à la fin comment les concours s’étaient passés.
(1) Alors là j’ai bien galéré 10 min avant qu’il me dise de considérer d’abord le cas n = 2
... On se rend compte que si A ∈ E (l’espace vectoriel qu’on cherche) alors Tr A = 0.
4 SPÉCIALE MP* : ORAL 2006

On conjecture donc E = {M ∈ Mn (R) | Tr M = 0}.


⊂ : évident par linéarité de la trace
⊃ : On traite donc d’abord le cas n = 2 pour y voir plus clair. A nilpotente ⇔
det A = Tr A = 0. Soit M de trace nulle :
       
a c 1 −1 0 1 0 0
M= =a + (a + c) +(b − a) .
b −a 1 −1 0 0 1 0
| {z } | {z } | {z }
nilpotente nilpotente nilpotente

Donc M ∈ E. On a bien E = {M ∈ Mn (R) | Tr M = 0} et cet espace est de dimension


3 car les 3 matrices précédentes sont libres et génératrices.
 
m1 ⋆
.. Pn
Cas général : c’est le même principe, on a M =  .  avec mi = 0. On
i=0
⋆ mn
va s’arranger pour avoir d’abord la diagonale : on a
 
  0 0 0 0  
1 1 0 0 1 −1  0 0
 −1 −1     

m1   +(m +m )  0 1 −1  + · · ·+(mn−1 +...+m1 ) 
 2 1    1 −1 
 
0 0 0 1 −1
| {z } 0 0 | {z }
A1 : nilpotente | {z } An : nilpotente
A2 : nilpotente:
 
m1 ⋆′
= .. 
.

⋆ mn
et comme la trace est nulle, mn = −(m1 + ... + mn−1 ) et on a juste besoin de n − 1
matrices pour former celle-ci (et cette famille est libre).
Ensuite on utilise les n2 − n matrices
 i Eij(i 6= j) qui sont nilpotentes, avec les bons
P P
n−1 P
coefficients pour obtenir M = mk Ai + βij Eij . On a donc M ∈ E et E est
i=1 k=0 i6=j
de dimension n2 − 1.
(2) Là j’ai eu de la chance car j’avais eu cet exo en colle avec Ginoux dans l’année donc j’ai
pu aller vite et me rattraper suiteau laborieux premier
 exo.
0 0 0 1 1 0
Avec A = et B = on a AB = 6= 0 et BA = 0. (En dimension
1 0 0 0 0 0
n, prendre A = Eij et B = Ejk avec k 6= i).
(3) Par l’absurde on suppose qu’il existe une norme N invariante par similitude. Soit
A ∈ Mn (R), ∀P ∈ GLn (R) on a N(A) = N(P AP −1 ). Avec B = AP on a N(AP ) =
N(B) = N(P BP −1 ) = N(P A). Donc ∀P ∈ GLn (R) on a N(AP ) = N(P A), or GLn (R)
est dense dans Mn (R), donc ce résultat reste vrai pour tout P ∈ Mn (R). Il suffit de
prendre les A et B du a. : N(AB) = 0 6= N(BA) d’où la contradiction.
(4) La trace est linéaire et la valeur absolue est une norme, de plus Tr M = 0 ; M = 0
donc on a bien les propriétés d’une semi-norme (vérification immédiate).
(5) Comme Tr AB = Tr BA, la semi-norme précédente est invariante par similitude. Pour
en trouver d’autres j’ai cherché les invariants pour des matrices semblables mais on
obtient rien, il peut être intéressant de penser en termes d’endomorphismes mais je n’ai
pas eu le temps de finir. Il m’a demandé ce que j’en pensais, j’ai dit qu’à mon avis
c’était la seule semi-norme mais depuis j’en doute...
SPÉCIALE MP* : ORAL 2006 5

Solution 1.2.3 (P. Adroguer) Note : 14


Examinateur : calme, serein, sympa, laisse passer les “arnaques” s’il voit qu’on a compris.
On peut déjà supposer a et A > 0.
On divise par x2 , à la limite quand x → +∞, on a a 6 A (si A = 0 alors |b| 6 |B| en divisant
par x et en faisant tendre x vers +∞).
Avec x = 0, on a |c| 6 |C|.
Posons P (x) = Ax2 + Bx + C et p(x) = ax2 + bx + c.
• Premier cas : si B 2 − 4AC = 0 alors P (x) admet une racine double x0 et comme
|p(x0 )| 6 P (x0 ) alors p(x0 ) = 0.
Montrons par l’absurde que x0 est racine double de p : en effet, si x0 n’est pas racine
double de p alors p′ (x0 ) 6= 0 donc, si par exemple p′ (x0 ) > 0 alors p(x) ∼ p′ (x0 )(x−x0 ) >
P (x) au voisinage de x0 .
Conclusion : x0 est racine double de p donc b2 − 4ac = 0.
• Si B 2 −4AC > 0, P a 2 racines x1 > x2 donc p admet les mêmes racines donc b2 −4ac > 0.
−B −b x1 +x2 B 2 − 4AC x1 +x2 b2 − 4ac
On a donc x1 + x2 = = et |P ( 2 )| = > |p( 2 )| =
A a 4A 4a
a
donc b2 − 4ac 6 (B 2 − 4AC) 6 B 2 − 4AC.
A
• Si B 2 − 4AC < 0 alors C > 0 et ∀x ∈ R, P (x) > 0, on a alors p(x) 6 P (x) et
−p(x) 6 P (x) soit
(A − a)x2 + (B − b)x + (C − c) > 0 et (A + a)x2 + (B + b)x + C + c > 0
et, en calculant les discriminants qui sont négatifs, on obtient
B 2 − 2bB + b2 − 4AC + 4aC + 4Ac − 4ac 6 0
B 2 + 2bB + b2 − 4AC − 4aC − 4Ac − 4ac 6 0
puis, en faisant la somme, on obtient b2 − 4ac 6 −(B 2 − 4AC) = |B 2 − 4AC|.
– Si b2 − 4ac > 0, c’est gagné.
– Si b2 − 4ac < 0 alors c > 0 et p(x) > 0 pour tout x. Or
B B b
min P (x) = P (− ) > p(− ) > min p(x) = p(− )
x∈R 2A 2A x∈R 2a
4AC − B 2 4ac − b2
soit > et on se retrouve dans la situation du deuxième cas.
4A 4a

Solution 1.2.4 (N. Charon) Note : ?


Examinateur : jeune et vraiment sympathique, il laisse chercher avant de donner quelques
indications bien placées, confirme lorsqu’on est sur la bonne voie et ne demande pas trop de
détails sur les arnaques qu’on peut lui faire, ce qui fait que l’oral est assez agréable. Par contre,
on dirait qu’il aime bien l’algèbre. Durée : 45 mn sans préparation.
(1) Au début, je voyais pas très bien quoi faire alors j’ai commencé par le cas simple où
r = n, i.e. M est inversible. Dans ce cas, A et B sont semblables et elles ont même
polynômes caractéristiques.
Pour le cas général, on commence par utiliser la sacro-sainte propriété :
∃(P, Q) ∈ GLn (R) telles que M = P Jr Q.
Puisque AM = MB, on obtient P −1 AP Jr = Jr QBQ−1 . On se ramène ainsi au cas où
M = Jr . On traduit ensuite le fait que AJr = Jr B en faisant
 le produit matriciel par
A1 A2 B1 B2
blocs. En écrivant que A = et B = , on trouve que : A3 = 0,
A3 A4 B3 B4
   
D A2 D 0
B2 = 0 et A1 = B1 = D d’où A = ;B= . Puis, on a : PA (X) =
0 A4 B3 B4
det(A − XIn ) = det(D − XIr ) det(A4 − XIn−r ) et idem pour B.
6 SPÉCIALE MP* : ORAL 2006

(2) Là, j’ai commencé par traiter le cas où A et B sont diagonalisables. Étant donné qu’elles
commutent, elles sont simultanément diagonalisables et comme les polynômes minimaux
de A et B sont les produits des (X −λ) avec λ vap, on obtient assez facilement le résultat
voulu. Mais en fait la démonstration générale ne marche pas comme ça. Ici, il m’a un
peu aidé en me disant “Est-ce que vous ne pourriez pas relier le degré de ΠA avec la di-
mension d’un certain espace ?”. Et là, je me suis subitement rappelé l’exo de Franchini
à Centrale où l’on montre que det ΠA = dim C[A], C[A] étant l’algèbre des polynômes
sur C engendré par A.
On écrit alors que d’une part C[AB] ⊂ C[A, B] donc dim C[AB] = deg ΠAB 6
dim C[A, B], d’autre part, tout polynôme de C[A, B] se décompose au moyen de po-
lynômes du type Ai B j avec i ∈ [[0, deg ΠA − 1]], j ∈ [[0, deg ΠB − 1]] (on peut faire la
division euclidienne de tout polynôme de C[A] par ΠA , idem pour B). Donc cette famille
de deg ΠA × deg ΠB éléments est génératrice et de ce fait : dim C[A, B] 6 deg ΠA . deg ΠB
et on a le résultat annoncé.
(3) On avait vu cet exo dans le cours mais honte à moi je m’en souvenais plus. Il m’a
donc aidé en me suggérant d’étudier d’abord le cas où par exemple A est inversible.
En écrivant la définition du polynôme caractéristique, on montre facilement que PAB
et PBA sont égaux puis en utilisant la densité de GLn (C) dans Mn (C), on étend par
continuité ce résultat.

Solution 1.2.5 (P. Néron) Note : ?


(1) Prenons le cas de la dimension 1 : 
on confond ici E et C. u(z) = (a + ib)z induit
a −b
l’endomorphisme de matrice (ur (1) = (a, b) et ur (i) = (−b, a)). On a ainsi
b a
det ur = a2 + b2 = | det u|2.
P
n
Passons à une dimension n quelconque : si u(ej ) = akj ek dans une base (e1 , . . . , en )
k=1
alors, dans la base (e1 , ie1 , . . . , en , ien ), on aura
X n
ur (ej ) = Re(akj )ej + Im(akj )iej
k=1
n
X
ur (iej ) = − Im(akj )ej + Re(akj )iej .
k=1

Or on sait que ur est trigonalisable sur C donc


 il existe unebase dans laquelle la matrice
λ1 ∗
de u est triangulaire supérieure : M(u) =  . .  donc la matrice de ur sera
.
0 λn
triangulaire par blocs :
 

A1
.. 

M(ur ) =  . 
0 An
 
Re(λk ) − Im(λk )
où Ak = .
Im(λk ) Re(λk )
On en déduit immédiatement que det ur = det A1 × · · · det An = |λ1 |2 . . . |λn |2 = | det u|2 .
(2) a) Par l’absurde : si [0, 1[ est homéomorphe à ]0, 1[, soit f l’homéomorphisme en ques-
tion. [0, 1[\{0} est connexe par arcs alors que ]0, 1[\{f (0)} ne l’est pas ce qui est
contradictoire.
SPÉCIALE MP* : ORAL 2006 7

b) Si f est un homéomorphisme de B(0, 1) ∪ {(1, 0)} alors on entoure f (0, 1) (qui est
dans la boule B(0, 1)) par un cercle C. f −1 (C) est aussi une courbe fermée mais
elle ne peut entourer (0, 1)...

Solution 1.3.1 (Th. Pradeau) Note : 12


Celui de la salle Résistances, très sympa, il avait l’air intéressé même s’il est sorti 2 ou 3 fois
pendant l’oral. Comme beaucoup d’examinateurs de l’ENS, il a terminé en me demandant en
quoi l’ENS m’intéresse et ce que je veux faire plus tard.
(1) On a lim P (X) = +∞.
X→+∞
Soit j = min {i | ai 6= 0}, au voisinage de 0+ on a P (x) ∼ −aj X j < 0 car aj > 0.
Comme P est continue, d’après le TVI on a bien une racine strictement positive.
(2) Par l’absurde ρ > ai0 + 1 en notant ai0 = max {ai }.
P i
n−1 P ai
n−1
P (ρ) = 0 donc ρn = ai ρ et ρn−i
= 1. Or ρ > ai0 + 1 6= 1 (ai0 > 0) donc
i=0 i=0  n
1
ai ai0 P
n
ai0 ai0
1− ai +1
1
ρn−i
6 n−i et 1 6 i = ai0 +1
0
1− a 1+1
=1− ai0 +1
. Contradiction.
(ai0 +1) i=1 (ai0 +1) i0
P
n−1 P
n−1
(3) P (λ) = 0 donc λn = ai λi et |λ|n 6 ai |λ|i . Donc P (|λ|) 6 0, or pour X > ρ on a
i=0 i=0
P (X) > 0 par définition de ρ. Donc on a |λ| < ρ.
P
n−1
(4) On va montrer que ρ n’est par racine de P ′ . P ′ (X) = nX n−1 − iai X i−1 donc ρP ′ (ρ) =
  i=0
P
n−1 P
n−1
nρn − iai ρi < n ρn − ai ρi = 0 car P (ρ) = 0. Donc P ′(ρ) 6= 0 ie ρ est racine
i=0 i=0
simple.
(5) Pas fait.

Solution 1.3.2 (P. Adroguer) Note : 14 et (V. Laviron) Note : ?


Examinateur : sympa, n’a pas dit grand’chose, juste ce qu’il fallait pour me débloquer.
(1) On cherche A diagonalisable, A = P Diag(λi )P −1 alors A2 = P Diag(λ2i )P −1 donc,
f i ∈ [[1, n]], ∃j ∈ [[1, n]] tel que λ2i = λj .
On suppose |λ1 | 6 |λ2 | . . . 6 |λn | alors |λ2n | 6 |λn | donc |λn | 6 1 puis |λ1 |2 6 |λ1 | donc
|λ1 | > 1.
Conclusion : ∀j ∈ [[1, n]], λj = eixj .
2j π
On cherche xj tel que x1 = 2xn −2π et xj+1 = 2xi , on mouline et on trouve xj = n .
  2 −1
1 1
(2) Dans GL2 (R), A = convient, dans GLn (R), il suffit de compléter par une
0 1
matrice identité.  
2 j 0
(3) On montre que dans GL2 (C) alors M est semblable à M ssi M est semblable à
0 j2
 
1 1
ou à .
0 1
 
a 0
Si M est diagonalisable (sur C) alors M est semblable à et on a a2 = b, b2 = a
0 b
ou bien a2 = a, b2 = b. On écarte le cas où M = I2 ce qui donne {a, b} =  {j, j 2 } 
dans
j 0
le premier cas, le deuxième cas est à rejeter. On a donc ici M semblable à .
0 j2
8 SPÉCIALE MP* : ORAL 2006
 
a 1
Si M n’est pas diagonalisable alors M est semblable à donc a2 = a soit a = 1
0 a
(a = 0 est à rejeter).
Réciproque : immédiate.
(4) Dans GL2 (R) alorsM estsemblable sur C àl’une des
 2 formes ci-dessus. Il suffit alors
j 0 0 1
de remarquer que est semblable à .
0 j2 −1 1
Conclusion :  les matrices
 de M2 (R) qui sont semblables à leur carré sont les matrices
0 1 1 1
semblables à ou à .
−1 1 0 1

Solution 1.3.3 (P. Néron) Note : ?


(1) Soit X = a2 + b2 et Y = c2 + d2 alors
XY = (a2 + b2 )(c2 + d2 ) = (ac − bd)2 + (ad + bc)2
(par analogie avec le module des nombres complexes) donc S2 est stable.
(2) S3 n’est pas stable car 3 = 12 + 12 + 12 , 5 = 12 + 22 + 02 mais 15 n’est pas une somme
de 3 carrés.
(3) S3 (Z) : on écrit les carrés dans Z/8Z : 0, 1, 4 puis les sommes de carrés : 0 1, 2, 3, 4,
5, 6... et on voit que 7 n’y appartient pas.
S3 (Q) : ?
(4) [0, 1] est compact alors que ]0, 1[ ne l’est pas.

Solution 1.4.1 (Th. Pradeau) Note : 12


Examinatrice : une femme assez cool, (souriante même avec le gars d’avant qui avait 30 min
de retard), apparemment fatiguée et pas très concentrée sur ce que je faisais : elle n’avait pas
vu que A−1 B et I commutent ni que det I = 1 !
(1) a) On peut écrire H = P DP T , donc
det(I + tH) = det(P T (I + tD)P )
= det(I + tD)
Yn
= (1 + λi t)
i=1
en notant λi les valeurs propres de H.
b) On a
((αA + (1 − α) B) x|x) = α(Ax|x) + (1 − α) (Bx|x) > 0
donc αA + (1 − α) B est symétrique définie positive et son déterminant est stricte-
ment positif. On peut donc prendre le logarithme (qui est croissant), il faut montrer
que
Xn
ln (det (αA + (1 − α) B)) > α ln(det A) + (1 − α) ln(det B) = α ln λi + (1 − α) ln µi
i=0

en notant λi et µi les valeurs propres (strictement positives) de A et B.


• Si A et B commutent, alors elles sont simultanément diagonalisables et par
Qn
conséquent det(αA + (1 − α) B) = (αλi + (1 − α)µi ). On a donc
i=1
P
n P
n
ln (det (αA + (1 − α) B)) = ln(αλi + (1 − α)µi) > α ln λi + (1 − α) ln µi
i=0 i=0
SPÉCIALE MP* : ORAL 2006 9

car le logarithme est strictement concave (les λi et les µi ne sont pas les mêmes
car A 6= B d’où la stricte inégalité). On a donc le résultat.
• Sinon on a det (αA + (1 − α) B) = (det A)(det(αI + (1 − α)A−1 B)) (A est
inversible car définie positive). Or A−1 B et I commutent et A−1 B 6= I donc
1−α
det (αA + (1 − α) B) > (det A) (det A−1 B) = (det A)α (det B)1−α .
Il doit y avoir une autre façon, parce-qu’on a pas utilisé le a...
c) J’ai commencé à parler de topologie en disant qu’il faut introduire une norme, mais
elle voulait juste que je lui dise que cet ensemble est convexe, ce qui est immédiat
vu ce qu’on vient de faire.
P
k P
k−1 P
k−1
(2) Alors on a uk = ui − ui = (ui+1 − ui) car u0 = 0. Donc avec Cauchy-Schwarz
i=0 i=0 i=0
P
k−1 P
j−1
on a u2k 6 (k − 1) (ui+1 − ui ) 2 6 j (ui+1 − ui) 2 pour k ∈ [|0, j|] . On obtient ainsi
 j−1i=0  i=0
Pj P
j P P
j−1 P uk+1 −uk 2
j−1
u2k leq j (ui+1 − ui )2 = j 2 (ui+1 − ui )2 = h
et voila.
k=0 k=0 i=0 i=0 k=0

Solution 1.4.2 (V. Laviron) Note : ?


Rt
(1) On pose F (t) = 0 φ(s) ds, on a tF ′ (t) 6 C + F (t). On résout l’équation différentielle
ty ′ = C + y qui donne y(t) = at − C. On pose alors G(t) = tF ′ (0) − C (qui vérifie
cette équation différentielle) et H = F − G. tH ′ 6 H d’où, en intégrant H(t) 6 bt et
H(0) = −C donc H(t) 6 |C| + bt soit F (t) 6 G(t) + |C| + bt. Ceci peut s’écrire plus
simplement F (t) 6 At + B.
On a ainsi t.φ(t) 6 C + At + B. On en déduit que φ est majorée sur R+ . Par contre,
on ne peut prouver que φ est bornée sur R+ , prendre φ(t) = −t donc il faut supposer
φ > 0 pour conclure.
(2) ?
Pn Pn
(3) On calcule X T MX = 2 x2i + 2 xi xi+1 (en posant xn+1 = x1 ). Or on peut réécrire
i=1 i=1
cette somme en faisant apparaı̂tre des carrés :

X T MX = (x1 + x2 )2 + (x2 + x3 )2 + · · · + (xn + x1 )2 > 0.

En plus X T MX = 0 ⇒ x1 + x2 = . . . = xn + x1 = 0. Si n est impair alors on trouve


x1 = . . . = xn = 0, i.e. A est définie positive. Si n est pair alors xk = (−1)k vérifie cette
relation donc A n’est pas définie.

Solution 1.4.3 (P. Adroguer) Note : 18


Examinateur : R.A.S. sinon qu’il a très peu apprécié la pléthore d’erreurs de calculs que j’ai
faites.

(1) On sépare les 2 sommes (n > 0 et n < 0) et on prouve qu’il y a convergence uniforme.
Grâce aux théorème de dérivation d’une série, on prouve que f est C ∞ . f est paire,
2π-périodique.
10 SPÉCIALE MP* : ORAL 2006
Z 2π
1
(2) On a c0 = f (t) dt soit
2π 0
Z 2π X
1 1
c0 = dt
2π 0 n∈Z a + (t − 2nπ)2
2

Z
1 X 2π dt
= grâce à la convergence normale
2π n∈Z 0 a2 + (t − 2nπ)2
Z
1 X (−2n+2)π dt
= en faisant un changement de variable
2π n∈Z −2nπ a + (t − 2nπ)2
2

Z +∞
1 dt
= en regroupant les intégrales
2π −∞ a + t2
2

1
= .
2a
Z Z +∞
2 π 1 cos pt dt
Par le même calcul, on trouve ap = f (t) cos pt dt = .
π 0 2π −∞ a2 + t2
Z
a cos pt 1 +∞
(3) On pose g(a, t) = 2 , ϕ(a) = g(a, t) dt. Grâce au théorème de Lebesgue
a + t2 2 −∞
a
de dérivation sous l’intégrale, on prouve que ϕ est C ∞ . Si h(a, t) = 2 , on trouve
a + t2
∆h = 0 donc
Z
′′ 1 +∞ ∂2h
ϕ (a) = cos pt 2 (a, t) dt
2 −∞ ∂a
Z +∞
1 ∂2h
= − cos pt 2 (a, t) dt
2 −∞ ∂t
et une double I.P.P. permet de trouver p2 ϕ(a) = ϕ′′ (a), h s’annulant à l’infini.
Les conditions ap → 2c0 quand p → 0 et ap → 0 quand p → +∞ permettent de trouver
ϕ.

Solution 1.4.4 (N. Charron) Note : ?


Examinatrice : jeune, plaisante et assez communicative pendant la colle. A ce que j’ai pu
comprendre, elle était en dernière année d’étude à Cachan. A la fin, elle demande ce qui nous
intéresse à Cachan et si on préfère aller à Paris ou à Ker Lan en Bretagne. Mais ce qui m’a
surtout plu, c’est qu’elle m’a posé le même exo qu’a eu Daniel Han Kwan l’an dernier (le 1.4.3)
que j’avais fait dix jours avant. En plus, il y avait davantage de questions intermédiaires.
(1) La démonstration se fait en deux parties et se base principalement sur des dessins!
∂f
• Unicité : L’hypothèse de départ sur f implique que > 0. Cela signifie que à x fixé,
∂y
l’application y 7→ f (x, y) est strictement croissante. Un dessin de la courbe f (x, y) =
0 nous permet alors de voir que l’unicité est assurée, sous couvert d’existence bien
évidemment.
∂f ∂f ∂f ∂f
• Existence : Un poil plus difficile, on écrit que > (1) et >− (2). (2)
∂y ∂x ∂y ∂x
∂f ∂f
s’écrit aussi + > 0, ce qui nous pousse à considérer la fonction x 7→ f (x, x)
∂y ∂x
qui est donc strictement croissante. Or f (0, 0) = 0 donc si x > 0 (le cas x < 0
est analogue), f (x, x) > 0. En considérant de même x 7→ f (x, −x), on trouve que
SPÉCIALE MP* : ORAL 2006 11

f (x, −x) < 0. Le T.V.I. appliqué à la fonction y 7→ f (x, y) permet donc de dire que
∃y|f (x, y) = 0.
A présent, une superbe fresque grandeur nature digne des plus grands peintres de la
Renaissance permet de récapituler ce qu’on vient de faire. Notamment, les droites y = x
et y = −x jouent un rôle capital.
Continuité : On commence par travailler en 0 : |g(x)| 6 |x| (si on ne voit pas ça, c’est
que le dessin a été mal fait!), donc c’est immédiat. Pour le cas x0 quelconque, on peut
se ramener par translation en 0, en considérant la fonction (x, y) 7→ f (x + x0 , y + g(x0)),
qui, ô joie, vérifie les mêmes hypothèses que notre chère fonction f . On en déduit que
|g(x) − g(x0 )| 6 |x − x0 |, d’où la continuité de g. On a prouvé en passant que g était
1-lipschitzienne.
Dérivabilité : Toujours en 0, la différentiabilité de f permet d’écrire que f (x, y) =
∂f
∂f ∂f − o(x, g(x))
0+x +y + o(x, y). Pour y = g(x), g(x) = x ∂x + . Il s’agit donc de
∂x ∂y ∂f ∂f
∂y ∂y
montrer que le deuxième terme est un o(x). Les normes étant équivalentes en dimension
finie, on peut choisir la norme infinie ||.||∞ et utiliser le fait que |g(x)| < |x|. En écrivant
que o(x, g(x)) = |x|h(x, g(x)), avec h → 0 pour x → 0, la conclusion semble immédiate,
sauf qu’il m’a pénibilisé en réclamant une démo rigoureuse, avec des ǫ, en justifiant sa
requête par un malhonnête ”Un jour quand ça sera plus compliqué tu te planteras!”.
Pour x0 quelconque, le même artifice que pour la continuité fournit le résultat.
(2) Comme tout bon malsain qui se respecte, j’ai plus ou moins fait le sujet d’études sur le
théorème d’inversion locale, donc je savais qu’il existait une démonstration utilisant ce
théorème.
La méthode employée pour cette planche est également applicable (on a les hypothèses
∂f
f est C 1 , f (0, 0) = 0, (0, 0) > 0 ou < 0, c’est pareil). Ici il m’a énormément guidé, je
∂y
me suis contenté de lui montrer que je comprenais ce qu’il qu’il me disait de faire.
Grosso modo, on a par continuité des dérivées partielles un rectangle contenant l’origine
∂f
où > 0. Il se trouve qu’on connaı̂t le signe de f sur une partie de l’axe des ordonnées :
∂y
pour |y| pas trop grand, f (0, y) > 0 si y > 0 et f (0, y) < 0 sinon. Il existe donc
par continuité un rectangle inclus dans le précédent où ∀x, ∃y + > 0, y − < 0 vérifiant
f (x, y + ) > 0 et f (x, y − ) < 0. D’ailleurs je viens de me rendre compte qu’il m’a bien
arnaqué, car l’existence de ce rectangle n’est pas si évidente que cela, il faut utiliser
Borel-Lebesgue (il y a peut-être plus simple). Grâce au T.V.I., le théorème des fonctions
implicites est démontré! Cela se généralise sans peine au cas n quelconque.

Solution 2.1.1 (R. Portalez) Note : 11


Barbu, fatigant pour le candidat et fatigué de lui-même. Il est complètement inerte mais
donne quand même quelques indications. Ne suit pas ce qui se passe au tableau, fait semblant
d’écouter puis après vous reprend quand il croit qu’on raisonne mal alors qu’il n’a pas suivi.
En clair, il vaut mieux rédiger tout au tableau vu que ce qu’on dit tombe dans l’oreille d’un
sourd. J’avais nettement préféré Langevin, quoiqu’en disent certains.
(1) Soit z une racine de P de partie réelle strictement positive. On ”remarque” que an + an−1 z
est de module plus grand que 1. (Pour démontrer ça il suffit de faire un dessin). Ensuite,
on écrit que P (z) = z n (an + an−1z
+ . . . + zan0 ). On en déduit que an + an−1
z
+ . . . + zan0
est nul et donc que an−2 z2
+ . . . + zan0 > 1. Si |z| 6 1 c’est terminé, sinon on a la série
d’inégalités suivantes :
12 SPÉCIALE MP* : ORAL 2006

an−2 a0
1 6 2
+ ...+ n
z z
1 1
6 9( 2 + . . . + )
|z| |z|n
1
6 9( 2 + . . .)
|z|
x2
= 9
1−x
En posant x = 1/|z| < 1. √
On en déduit que 1 − x 6 9x2 et, comme x est un réel positif, que x > −1+18 37 . En
passant à l’inverse, on trouve le résultat voulu.
P
n
(2) Si x est un vecteur propre associé à la valeur propre λ alors mij xj = λxi . On fait
j=1
alors la somme sur i :
n X n n n
! n
X X X X
mij xj = mij xj = xj
i=1 j=1 j=1 i=1 j=1
Xn
=λ xi
i=1

d’où λ = 1.  
1
Si la somme des termes de chaque ligne vaut 1 alors le vecteur  ...  est vecteur propre.
1
À partir de là, on peut dire beaucoup de choses :
P
n
• Si M stabilise l’hyperplan xi = 0 alors 1 est valeur propre simple (le vecteur
i=1
propre que l’on vient de proposé est en somme directe avec l’hyperplan).
• Si M a tous ses coefficients positifs, on peut aussi montrer que 1 est valeur propre
simple et que toutes les autres valeurs propres sont de module < 1.
• Il doit y avoir d’autres propriétés...

Solution 2.1.2 (Y. Vézine) Note : 8


Commentaire : Ce type est un déchet de l’humanité : son attitude durant la colle est plus que
passive, il ne parle pas beaucoup et je le soupçonne même de s’être endormi au milieu de la colle
(je l’ai surpris les yeux fermés et la tête appuyée sur ses mains...). Il a même eu le culot de me
dire que j’étais trop ”excité” (je pense que le moindre enthousiasme de la part d’un candidat
l’empêche de dormir). En gros il faut faire comme s’il n’était pas là, ce qui n’est ni facile ni
agréable, et rend la colle plus que pénible alors que l’exo est somme toute assez intéressant (ce
qui est assez rare avec Grigis).
• D’abord histoire de se sentir moins seul on donne quelques exemples de fonctions de Φ :
sinus, cosinus, exponentielle, polynômes,...
• Une fois qu’on a gagné un peu de temps on se dit que c’est pas pour rien que les fonctions
sont C ∞ , et après avoir pris f ∈ Φ, on considère f ′ . On va montrer que f ′ ∈ Ff . Soit
(fj ) une base de Ff . Pour x ∈ R, h ∈ R, on peut écrire :
f (x + h) − f (x) X
= aj (h)fj (x)
h
SPÉCIALE MP* : ORAL 2006 13

Le membre de gauche tend vers f ′ (x) quand h → 0, et comme Ff est fermé (dimension
h→0
finie), on peut dire que ∀j, aj (h) → bj ∈ R, d’où l’on déduit que f ′ ∈ Ff .
• On a ainsi Ff ′ ⊂ Ff , et donc f ′ ∈ Φ. Ainsi Ff ⊃ Ff ′ ⊃ · · · ⊃ Ff (k) ⊃ · · · . Comme Ff
est de dimension finie, on peut affirmer que f est solution d’une équation différentielle
linéaire homogène à coefficients constants.
• A partir de là je ne vois pas trop ce que l’on peut faire de plus pour caractériser Φ.
Apparemment l’exo est déjà tombé à l’oral car Bogdan l’a déjà fait et le candidat juste
après moi aussi. Heureusement pour lui Grigis pose deux fois de suite le même exo donc
cette petite ordure a bien torché en faisant semblant de tout inventer (ce qui est toutefois
assez légitime). Il m’a dit que Φ est l’espace engendré par les produits d’exponentielles
par des polynômes. Affaire à suivre.
Soit an y (n) + an−1 y (n−1) + · · · + a0 y = 0 l’équation différentielle en question.
– On cherche tout d’abord les solutions de la forme y = erx , r doit alors satisfaire
l’équation résolvante :
P (r) = an r n + an−1 r n−1 + · · · + a0 = 0.
Si r est racine d’ordre p > 1 alors y = xq erx est aussi solution (pour q 6 p − 1) :
y = xq erx
X k  
(k) k q!
y = xq−l r k−l erx
l=0
l (q − l)!
X n  
(n) n q!
y = xq−l r n−l erx
l=0
l (q − l)!
 
k
avec la convention classique, = 0 si l > k. On en déduit que
l
n n k  
!
X X X k q!
ak y (k) = ak xq−l r k−l erx
l (q − l)!
k=0 k=0 l=0
n n   !
X q! X k
= xq−l ak r k−l erx
l=0
(q − l)! k=0
l
=0
car
n
X   n
k k−l 1X k!
ak r = ak r k−l
k=0
l l! k=0
(k − l)!
1 (k)
= P (r) =0
l!
vu que r est racine d’ordre p > q de P . On remarque ensuite que la famille (xq erx )
est libre, r racine d’ordre p et q < p. Comme l’ensemble des solutions est un espace
vectoriel de dimension n, on a affaire à une base.
Conclusion : l’ensemble des solutions de l’équation an y (n) +an−1 y (n−1) +· · ·+a0 y = 0
Pj
s’écrit y = λi Pi (x)eri x où ri est racine d’ordre pi de la résolvante et où deg Pi < pi .
i=1

Solution 2.1.3 (V. Laviron) Note : 7


14 SPÉCIALE MP* : ORAL 2006
Z T
x
(1) On fait une intégration par partie avec eit dt :
1
itx x−1 x
du = e ixt dt u = eit
−i 1−x −i
v= t dv = (1 − x)t−x dt
x x
Z T  T Z T itx x
itx −i 1−x itx 1−x e eit
d’où e dt = t e +i dt. Or g(t) = x est intégrable sur
1 x 1 x 1 tx t
1
[1, +∞[ (|g(t)| = x et x > 1.
Z t T
x
Conclusion : eit dt admet une limite quand T → +∞ et
1
Z 1 Z +∞ x
itx i 1−x eit
f (x) = e dt + ei +i dt.
0 x x 1 tx
itx x itx
(2) Sur [0, 1[, e → 1 car t → 0 et e = 1 qui est intégrable donc la version continue du
théorème de Lebesgue s’applique (utiliser le critère séquentiel d’existence d’une limite)
R1 x
donc lim 0 eit dt → 1.
x→+∞
x
| eit | 1
Sur [1, +∞[, x
= x donc
t t
Z +∞ itx Z +∞
e 1
x
dt 6 t−x dt = →0
1 t 1 x−1
donc lim f (x) = 1.
x→+∞

Solution 2.1.4 (J. Assoun) Note : 12


Examinateur : muet ou presque. J’étais son premier candidat pour la session 2006 et il semblait
déjà en avoir marre en donnant l’exercice. Il m’a fait calculer les coefficients de Fourier (!).
Ensuite, il semblait s’attendre à ce que je résolve l’équation différentielle (encore des calculs)
et a eu l’air plus intéressé lorsque que j’ai proposé ma solution, ce qui ne l’a pas empêché de
me demander quand même la résolution (j’ai fait la variation de la constant sans me souvenir
du cours de sup...).
Je trouve la note assez sévère dans la mesure où j’ai fait son exo (même si ce n’était pas sa
méthode).
(1) Calculs... On peut cependant remarquer que g est π-périodique, paire, continue et de
classe C 1 par morceaux (ce qui permet d’anticiper sur la forme du résultat.
(2) On peut utiliser la méthode du cours qui consiste à chercher une solution développable
en série de Fourier mais ce n’est pas une bonne idée ici. En effet, on résout y ′′ +y = sin x
sur [0, π] et y ′′ + y = − sin x sur [−π, 0]. En fait on cherche les solutions complexes de
z ′′ + z = ± eix sous la forme z = P (x) eix où P est un polynôme et on prend la partie
imaginaire. On trouve alors (avec relativement peu de calculs)

y = − x cos x + λ cos x + µ sin x si x ∈ [0, π]
2 . La condition de continuité en 0
y = x cos x + λ′ cos x + µ′ sin x si x ∈ [−π, 0]
2
1 1
donne λ = λ′ , y ′ continue en 0 donne − + µ = + µ′ , y ′′ = −y + | sin x| donne
2 2
′′ π
la continuité de y . Enfin, la condition y(0) = y(π) entraı̂ne λ = et y ′(0) = y ′(π)
4
1
donne µ = .
2
SPÉCIALE MP* : ORAL 2006 15

Conclusion : la seule solution du problème est donnée par


 
π |x| 1
y(x) = − cos x + | sin x|, x ∈ [−π, π]
4 2 2
et on vérifie qu’elle convient.
Remarque : on peut penser qu’il y avait d’autres questions à la suite car comment
interpréter les deux premières questions autrement ?

Solution 2.1.5 (P. Néron) Note : 10


(1) On écrit n = qk + nk où q est le quotient de n par k. Pour un cas favorable, on a
k
qk + 6 n = qk + nk < qk + k = (q + 1)k
2
n 2n
soit <k6 .
q+1 2q + 1
n n n
• k> est équivalent à k > + 1 : en effet, si est entier alors c’est
q+1 q+1 q+1
n n n n
immédiat et si n’est pas entier, on a E( q+1 ) < q+1 < E( q+1 ) + 1 6 k.
q+1  
2n 2n
• k6 est équivalent à k 6 E .
2q + 1 2q + 1
Pn
2n n
Le nombre de cas favorables est donc E( 2q+1 ) − E( q+1 ). On suppose implicitement
q=1
que les probabilités sont équidistribuées, on en déduit :
n  
1X 2n n
pn = E( ) − E( )
n q=1 2q + 1 q+1
1 2n 1 n 2 1
(2) On pose an,q = E( 2q+1 ) − E( q+1 ). lim an,q = − . Il n’est pas facile de
n n n→+∞ 2q + 1 q + 1
majorer an,q par le terme d’une série convergente indépendante de n, donc on va faire
la démonstration à la main :
2n 2n n 2n
• Tout d’abord comme 6 = on a E( 2q+1 ) 6 E( nq ) donc
2q + 1 2q q
    
1 n n
0 6 an,q 6 E −E
n q q+1
P
n Pn 1 h    i 1 
d’où an,q 6 E nq − E q+1 n
= E Nn+1 (car les termes se
q=N +1 q=N +1 n n
1
télescopent et an,n = ).
n
P
N PN 2 1
• lim an,q = − qui est le terme d’une série convergente car
n→+∞ q=1 q=1 2q + 1 q+1
 
2 1 1 1
− = =O . On peut calculer la somme de cette
2q + 1 q + 1 (2q + 1)(q + 1) q2
série :
+∞
X +∞
X
2 1 2 2
− = −
q=1
2q + 1 q + 1 q=1
2q + 1 2q + 2
+∞
X (−1)p+1 2
= = 2 ln 2 − 1
p=3
p
16 SPÉCIALE MP* : ORAL 2006
  

 P
N 2 1 ε
 2 ln 2 − 1 − − 6
On choisit alors N assez grand pour que q=1 2q + 1 q + 1 3

 1  1 ε
 E Nn+1 6 6
n N +1 3
P
N 2 1 ε
puis n > N tel que an,q − + 6 .
q=1 2q + 1 q + 1 3
Alors
N
X   N
X
2 1 1 n 2 1
|2 ln 2 − pn | 6 2 ln 2 − 1 − − + E + an,q − −
q=1
2q + 1 q + 1 n N +1 q=1
2q + 1 q + 1

et OUF on peut conclure : lim pn = 2 ln 2 − 1.


n→+∞
(3) On utilise l’encadrement
Xn Xn
1 1 1 1
+ <e< +
k=0
k! (n + 1)! k=0
k! n.n!
n
X 1 1 1 1
donc n! + < n!e < An + . On a alors n!e − An ∼ et on peut conclure
k! n + 1 n n
| k=0
{z }
=An ∈N
que un ∼ n sin( 2π
n
) → 2π.

Solution 2.2.1 (Th. Pradeau) Note : 14


M. Henry, très sympa, assez ouvert sur des démonstration différentes des siennes, écrit tout sur
son carnet pendant l’oral. Il a décidé de renouveler ses exos pour moi ! (cf. ceux de Quentin,
Bogdan et Poitou).
 
0 1 0
 ... ... 
 
(1) On peut écrire la matrice de u dans la base canonique : M =  .  qui est
 . . 1
0 0
nilpotente. Quand on regarde de plus près on se rend compte qu’on peut conjecturer
que les sous-espaces stables par u sont les Fk = Vect(e1 , ..., ek ), pour k ∈ [|1, n|].
Il est d’abord évident qu’ils sont bien stables par u.
Soit F un sous-espace stable par u de dimension > 1.
Pn P
n−1
• Si x = xi ei ∈ F alors u(x) = xi+1 ei et, par une récurrence immédiate,
i=1 i=1
P
n−p
up (x) = xi+p ei (p 6 n − 1).
i=1
• Si x 6= 0, soit p0 est l’indice maximal tel que xp0 6= 0 alors
n−p0 +1
X
up0 −1 (x) = xi+p0 −1 ei = xp0 e1 ∈ F
i=1

donc e1 ∈ F car xp0 6= 0, de même up0 −2 (x) = xp0 e2 + xp0 −1 e1 ∈ F donc e2 ∈ F . Par
une récurrence (encore immédiate) on montre que ei ∈ F pour i ∈ [[1, p0 ]].
SPÉCIALE MP* : ORAL 2006 17

• Soit alors k l’indice maximal tel que ek ∈ F , nécessairement, tout vecteur x ∈ F


P
k
s’écrit x = xi ei soit F ⊂ Fk . Mais comme on vient de voir que les (ei ) pour i 6 k
i=1
sont dans F , on a l’inclusion dans l’autre sens.
Conclusion : seuls les Fk sont stables par u.
(2) Alors là c’est étrange car il a dit ”très bien” à la fin du 1er exo (malgré l’arnaque
qui a été réparée) mais il m’en donne un second assez facile... On traduit que f est
lipschitzienne :
∃k > 0 | ∀(x, y) ∈ R2 , |f (x) − f (y)| 6 k|x − y|.
Prenons x > y alors k(y − x) 6 f (x) − f (y) 6 k(x − y) soit encore
0 6 f (x) + kx − (f (y) + ky) 6 2k(x − y).
Donc u = f +k Id vérifie u(x) > u(y) i.e. est croissante, tout comme la fonction v = k Id .
On a bien f = u − v.
(3) On traduit l’uniforme continuité :
∀ε > 0, ∃η > 0 | ∀(x, y) ∈ [0, 1[2, |x − y| 6 η ⇒ |f (x) − f (y)| 6 ε.
f vérifie le critère de Cauchy d’existence de limite en 1 donc on peut la prolonger par
continuité sur [0, 1] et le prolongement de f est borné, il en est de même de f .
(4) Par l’absurde ∃ [a, b] | ln x = P (x) pour x ∈ [a, b] (a > 0) alors x1 = P ′ (x) soit xP ′ (x) = 1
pour x ∈ [a, b] donc xP ′ (x) − 1 est le polynôme nul. On raisonne alors sur les degrés :
deg XP ′ > 1 et deg 1 = 0 d’où la contradiction.

Solution 2.2.2 (B. Dobre) Note : 16


(1) wn est une intégrale de Wallis, cf. cours.
(2) a) Par l’absurde, supposons que P admette une racine double.
Soit D = pgcd(P, P ′). deg(D) > 1 car P admet racine double. Or P = DR,
avec D et R dans Q[X] (en utilisant l’algorithme d’Euclide) Comme deg(D) > 1 et
deg(R) > 1 (car deg(D) 6 deg(P ′)), P réductible, contradiction!
b) P réductible dans Q[X], d’après a). Soit P = T S, avec deg(T ) 6 deg(S), T et S
dans Q[X].
Si deg(T ) = 1, c’est bon.
Sinon, deg(T ) = 2 et deg(S) = 3.
Si T racine double, OK. Pareil pour S.
Sinon, T et S ont une racine commune donc D = pgcd(T, S) de degré > 1. T = DF
et S =PDH. Selon le degré de D(1 ou 2), F ou H de degré 1 donc racine dans Q !
(3) Soit Sn = nk=1 xk .
n
X n−1
X n−1
X
ak xk = an Sn + (ak − ak+1 )Sk > (ak − ak+1 )Sk .
k=1 k=1 k=1
P
Donc (ak − ak+1 )Sk majorée a termes positifs. Elle converge.
En revenant a l’égalité ci-dessus, an Sn converge.

Solution 2.3.1 (Th. Pradeau) Note : 9


Examinateur sympa, marche beaucoup et ne note rien contrairement à Henry, l’oral ressemble
plutôt à un dialogue. En revanche il ne connaı̂t pas le programme (et donc ce qu’on n’est pas
censé savoir), mais est intéressé par ce qu’on fait. Même si au début on se dit qu’il n’y a pas
de géométrie eh bien il y en a quand même !
18 SPÉCIALE MP* : ORAL 2006

(1) On commence par  le cas n = 2, ce qu’il a apprécié :


a b
avec A = on a (ad − bc)2 6 (a2 + b2 )(c2 + d2 ) car (ac + bd)2 > 0. Il y a égalité
c d
ssi ac + bd = 0 i.e. si A est orthogonale.
Cas général : Première idée : faire une récurrence avec les cofacteurs (il avait l’air
étonné que je connaisse la comatrice!) en développant selon la première ligne, mais ça
marche pas (il a pris une feuille pour faire le calcul en même temps que moi !) car
!2 n
! !
Pn X Pn
(det A)2 = a1j det A′1j 6 a21j (det A′1j )2 avec Cauchy-Schwarz. D’où
j=1 j=1 j=1
| {z }
||C1 ||2
Q
n Q
n
comme det A′1j 6 ||Ci′ || 6 ||Ci|| d’après l’hypothèse de récurrence, avec Ci′ les co-
i=2 i=2
Q
n
lonnes de la comatrice, on obtient | det A| 6 n ||Ci ||.
i=1
Deuxième idée : on sait que A est trigonalisable. Si elle est triangulaire le résultat
est immédiat (le terme de la diagonale est inférieur à la norme de sa colonne), sinon
T
on écrit A = P T P ⋆ (avec P ⋆ = P ). On considère alors AA⋆ , dont les termes diago-
naux sont les ||Ci||2 (il suffit de faire le produit pour s’en rendre compte), et le produit
des termes diagonaux donne donc le produit des normes au carré. Ensuite on utilise
la décomposition A = UT avec U unitaire et T triangulaire (je ne connaissais pas ce
résultat et je ne sais pas si le T est le même que celui de la décomposition A = P T P ⋆).
Alors AA⋆ = T T ⋆ et en prenant le déterminant on doit avoir le résultat (d’ailleurs ça
donne une égalité ???).
Troisième idée : Il faut savoir que | det A| est par définition le volume du parallélépipède
construit sur les n vecteurs C1 , ...Cn (ce qu’on savait déjà en dimensions 2 et 3).
Qn
Si la matrice est orthogonale | det A| = ||Ci|| (penser géométriquement : les Ci
i=1
sont orthogonaux) et le but est donc de montrer que le volume est maximal dans
ce cas. On prend det A 6= 0, sinon c’est débile, et les (Ci ) forment une base
qu’on rend orthogonale avec l’algorithme de Schmidt : on obtient une base (Cei ) avec
Cei = Ci − pVect(Cf1 ...Cg
i−1 )
(Ci ). On remplace Ci par une combinaison linéaire de vec-
teurs où Ci apparaı̂t avec un coefficient 1, donc on ne change pas le déterminant :
Q
n Qn   Q
n
(det A)2 = ||Cei ||2 = ||Ci||2 − ||p f g (C i )||| 2
6 ||Ci||2 car ||Ci||2 =
Vect(C1 ...Ci−1 )
i=1 i=1 i=1
ei||2 + ||p
||C f1 ...Cg (Ci )|||2 avec Pythagore. On a ainsi le résultat voulu.
Vect(C i−1 )
(2) Si un+2 = aun+1 + bun avec a + b = 1 alors un+2 + bun+1 = un+1 + bun = u1 + bu0 .
u1 + bu0
• Si b 6= −1 alors l(1 + b) = u1 + bu0 donc l = .
1+b
• Si b = −1 alors un = α + βn (1 est racine double de la résolvante) d’où β = 0 et
l = u0 .
Si un+3 = aun+2 + bun+1 + cun avec a + b + c = 1 alors, en procédant comme au cas
précédent,
un+3 + (b + c)un+2 + cun+1 = u2 + (b + c)u1 + cu0 .
u2 + (b + c)u1 + cu0
• Si 1 + b + 2c 6= 0 alors l = .
1 + b + 2c
• Si 1 + b + 2c = 0 alors un vérifie la relation de récurrence un+3 − (2 + c)un+2 + (1 +
2c)un+1 − cun = 0 de résolvante r 3 − (2 + c)r 2 + (1 + 2c)r − c = 0 qui se factorise en
(r − 1)2 (r − c) = 0.
SPÉCIALE MP* : ORAL 2006 19

– Si c 6= 1 alors un = α + βn(+ γcn donc β = 0 et soit γ = 0 soit cn → 0


cu0 −u1
c−1
si |c| < 1
finalement un → α avec α =
u0 si |c| > 1
– Si c = 1 alors un = α + βn + γn2 d’où β = γ = 0 et un = u0 .

Solution 2.3.2 (Q. Chenevier) Note : 8


Si on identifie le plan à R2 , on peut dire que f est continue de R6 dans R2 .
f n’est pas surjective (pas étonnant) (0, −1, 0) n’a pas d’antécédent.
f n’est pas injective (f est définie à une isométrie près).
On peut alors fixer A (et le prendre comme origine), prendre B de coordonnées (x, 0) et C de
coordonnées (y, z) et définir f˜(A, B, C) = (xy, −x(y − x), −y(x − y) + z 2 ). f˜ est une application
C ∞ de R3 dans R3 qui n’est ni injective ni surjective...

Solution 2.3.3 (B. Dobre) Note : 16


On prend a, b, c, d les coefficients qui annulent la combinaison linéaire des colonnes du
déterminant.
• Si a + b + c + d 6= 0, on prend le barycentre de A, B, C, D avec les poids a, b, c, d et
on étudie l’application f (M) = aMA2 + bMB 2 + cMC 2 + dMD 2 (on remarque que
f (A) = f (B) = f (C) = f (D) = 0 compte tenu de la nullité de la combinaison linéaire).
−−→ −−→2 −−→ −−→2 −−→ −−→2 −−→ −−→2
f (M) = a GM + MA + b GM + MB + c GM + MC + d GM + MD
−−→ −−→ −−→ −−→ −−→ 
= (a + b + c + d)GM 2 + 2GM a | MA + bMB +
{z c MC + d MD}
=0
+ aAG2 + bBG2 + cCG2 + dDG2
donc
(a + b + c + d)GA2 = −[aAG2 + bBG2 + cCG2 + dDG2 ]
= (a + b + c + d)GB 2 = (a + b + c + d)GC 2 = (a + b + c + d)GD 2
par
r conséquent les points A, B, C, D sont sur le cercle de centre G et de rayon
aAG2 + bBG2 + cCG2 + dDG2
− .
a+b+c+d
−−−→ −−→ −−→ −−→ −−→ −−−→ −−→
• Si a + b + c + d = 0 on étudie g(M) = aMA + bMB + cMC + dMD. g(M) − GN =
−−→ −

(a + b + c + d)MN = 0 donc g est constante, on note K cette valeur. On reprend alors
le calcul de f (M) et cette fois-ci, on trouve
2 −
→ −−→
f (M) = aAO
| + bBO 2 {z
+ cCO 2 + dDO}2 +2 K .OM
=f (O)

où O est un point que l’on peut choisir arbitrairement.



→ − → −
→ −→ −→
– Si K 6= 0 alors f (A) = f (B)+2 K .BA (en prenant O = B), on en déduit que BA ⊥

→ −−→ −−→
K . De même avec BC, BD donc les points A, B, C, D sont alignés (éventuellement

→ −→ −→ −−→
confondus). Or, si les points ne sont pas confondus, K = bAB + cAC + dAD est un


vecteur directeur de la droite passant par les 4 points et K est orthogonal à cette
droite. On obtient une impossibilité donc dans ce cas, les 4 points sont confondus


mais alors K = 0, ce cas est à écarter définitivement.


– Il ne reste donc que le cas où K = 0
20 SPÉCIALE MP* : ORAL 2006

Solution 2.3.4 (F. Germain) Note : 14


L’examinateur n’est pas cité sur la feuille que m’a remise F. Germain mais on peut en deviner
sa marque.
(1) On vérifie immédiatement que α = c − b, β = a − c et γ = b − a conviennent et
f (z) = −αβγ.
Par translation et rotation, on se ramène au cas où A, B, C sont situés sur la droite réelle
et on appelle a, b, c leurs affixes respectives (qui sont des réels). Si M est d’affixe z alors
−−→ −→ −→
F (M) = MA2 BC + MB 2 CA+ MC 2 AB 2 2 2
h−−→=i |z −a| (c −b) + |z −b| (a−c) + |z −c| (b−a).
Or |z − a|2 = |z|2 − (z + z)a + a2 et BC = c − b d’où
  
F (M) = α |z|2 − (z + z)a + a2 + β |z|2 − (z + z)b + b2 + γ |z|2 − (z + z)c + c2
= (α + β + γ)|z|2 − (αa + βb + γc)(z + z) + αa2 + βb2 + γc2
= αa2 + βb2 + γc2 = −αβγ
car on a vu que α + β + γ = 0, αa + βb + γc = 0.
(2) On suppose p1 < p2 , on pose p = p1 et p2 = p + k alors
Ap+k = Ap ⇒ Ak Ap+k = Ap+2k = Ak Ap = Ap+k = Ap
et par une récurrence immédiate, Ap+nk = Ap . On pose alors B = Ap , n = p d’où
B k+1 = B.
P (X) = X k+1 − X est un polynôme annulateur de B qui a toutes ses racines simples
donc B est diagonalisable.

Solution 2.4.1 (R. Portalez) Note : 11


Sympa, il a rigolé quand j’ai allumé par mégarde le rétroprojecteur qui était dans la salle en
m’appuyant sur le bureau. Sinon il est barbu et assez jeune (une grosse trentaine). Comme l’a
dit Arnaud l’an dernier, il laisse mariner le candidat pendant 5-10 minutes puis il commence à
parler un peu. J’ai l’impression qu’il est content quand on a des idées, même si elles servent
pas à grand chose.
(1) Ma méthode On commence par remarquer qu’on peut supposer P inversible car les
coefficients pii sont tous positifs. Ensuite on suppose quePpii = 1 pour tout P i. Comme
P est diagonalisable sur R, on a det P = Πλi , et n = pii = Tr P = λi . Là on
se dit qu’il yPa anguille sous roche. On prend donc le logarithme du déterminant :
ln(det P ) = ln(λi ). On divise par n puis on utilise la concavité du log pour avoir
1
n
ln(det P ) 6 0. Cela montre le résultat dans ce cas particulier.
Dans le cas général, on fait un changement de base pour le produit scalaire que définit
P , de sorte que la nouvelle matrice ait des coefficients diagonaux unitaires. (On prend

Q = Diag(1/ pii )). Alors P ′ = QT P Q est symétrique positive et on peut lui appliquer
le premier cas. Et là : pouf, pastèque!
(2) La méthode qu’il m’a proposée à la fin P définit un produit scalaire. Grâce à Gramm-
Schmidt, on orthonormalise la base canonique avec une matrice de passage triangulaire.
On obtient P = QT Q, avec Q une matrice inversible triangulaire. Et là, en calculant
le produit, pii 6 αii2 (où Q = ((αij ))). Puis det P = (det P )2 = Παii2 6 Πpii et Pouf,
pastèque encore!
.
SPÉCIALE MP* : ORAL 2006 21

Solution 2.4.2 (Y. Vézine) Note : 14


Commentaire : Examinateur très sympathique, le seul examinateur que j’ai eu et qui m’ait
fait une colle digne de ce nom. Le principe est simple. Première partie : il écoute ce que le
candidat a à lui dire en acquiesçant tant que c’est OK, en demandant une précision ou deux en
cas d’arnaque ou de vanne, et en guidant un peu quand on part sur une piste sordide. Comme
ça il voit ce qu’on a dans le ventre, tout en testant la réactivité.
Deuxième partie : cinq minutes avant la fin, il te file un gros coup de pouce pour désosser et
rosser l’exo (d’où son nom). En gros la colle se fait à la bonne franquette, comme les colles que
l’on fait pendant l’année avec Jimmy ou certains autres colleurs.
(1) C’est très classique, je l’avait même eu en colle Jimmy deux semaines avant cet oral...
• Les Pj (X) forment une base de R[X] (c’est la base de Hilbert), donc on peut déjà
P
écrire P (X) = k0 aj Pj (X) où les aj sont des réels. Reste à montrer que les aj sont
entiers.
• On considère ∆ : P (X) → P (X + 1) − P (X). On remarque que pour j ∈ N et
p 6 j on a ∆p (Pj )(X) = Pj−p (X). En appliquant ∆k à P (X) et en faisant X = n
on obtient alors ak ∈ N (il faut remarquer que les Pj prennent des valeurs entières
sur Z).
• A partir de là on montre facilement le résultat souhaité par récurrence sur k, en
considérant à chaque fois le polynôme P (X) − ak Pk (X).
(2) Là c’est moins classique mais le principe est le même.
• On écrit R(X) = PQ[X] (X)
. En faisant la division de P par Q on obtient R(X) =
S(X) + r(X) où S est un polynôme (soit k son degré) et r une fraction rationnelle
de degré < 0. On veut montrer que r(X) = 0.
• On applique ∆k : ∆k (R)(X) = ak + ∆k (r)(X), en notant aj le coefficient d’ordre j
de S dans la base de Hilbert.
n→∞
• Or, ∆k (r)(n) → 0, et ∀n ∈ N, ∆k (R)(n) ∈ Z. Donc ak ∈ Z (Z est fermé). On
montre ainsi par récurrence sur k que les aj sont tous entiers.
• On en déduit que r(Z) ⊂ Z. Or, r est continue, tend vers zéro, et Z est discret,
donc r(X) = 0. Donc R est bien un polynôme (Rosso a même fait preuve d’humour
en me demandant si R pouvait être ”mal” un polynôme...).

Solution 2.4.3 (V. Laviron) Note : 13.5


Tout repose sur un coup de bluff ! L’exercice est très classique mais il peut faire mal.
On pose qC et qB les formes quadratiques de matrices C et B.
Comme A est symétrique, on sait que ∃P ∈ O(n) : A = P T DP où D = Diag(λi ), λi > 0 et en
Pn
faisant le produit des trois matrices on obtient aij = λk pki pkj .
k=1
P
n P P
Avec −

x = xi −

e i , on a qC (−

x ) = aij bij xi xj = λk pki pkj bij xi xj .
i=1 i,j i,j,k
P
n P
n
On pose alors −

xk = pkixi −

ei donc qC (x) = λk qB (xk ) > 0 ce qui donne le premier résultat.
i=1 k=1
Si A > 0, montrons que qC est définie :
qC (x) = 0 ⇔ ∀k ∈ [1, n], qB (xk ) = 0 car λk > 0
⇔ ∀k ∈ [1, n], −

x =0 k car qB est définie
⇔ ∀(k, i) ∈ [1, n]2 , pki xi = 0 car (ei ) est une base
⇒ PX = 0 où P est la matrice de passage
⇒x=0
22 SPÉCIALE MP* : ORAL 2006

ce qui montre le deuxième résultat.


Pour le troisième, je n’y ai pas encore réfléchi.

Solution 2.4.4 (J. Assoun) Note : 14


Examinateur sympathique mais son exercice est très difficile lorsqu’on n’a jamais utilisé la
différentielle seconde. Généreux sur la note. L’examen du cas n = 1 m’a permis de voir qu’il
fallait utiliser la formule de Taylor-intégral à l’ordre 1.
La formule de Taylor donne
Z 1
f (x) = (1 − u)f ′′ (ux)(x, x) du
0

2
 
∂ f
où f ′′ (v) est une forme bilinéaire symétrique de matrice H(v) = (v) (appelée matrice
∂xi ∂xj
R 1
hessienne de f ). On a donc f (x) = xT h(x)x où h(x) = 0 (1 − u)H(ux) du.
R 1 ∂2f
Il reste à montrer que h est de classe C 1 : il suffit donc de prouver que hij (x) = 0 (ux) du
∂xi ∂xj
est de classe C 1 . Pour cela, on utilise le théorème de dérivation sous le signe intégral, si x ∈
∂3f
B(0, R) qui est un compact de Rn alors ux ∈ B(0, R). On pose M = sup (v)
i,j,k,v∈B(0,R) ∂xi ∂xj ∂xk
∂3f ∂3f
alors (1 − u)u (ux) 6 M et (u, x) 7→ (ux) ∈ C([0, 1]×B(0, R)). On peut
∂xi ∂xj ∂xk ∂xi ∂xj ∂xk
donc appliquer ce théorème et conclure.

Solution 2.4.5 (P. Néron) Note : 10.5


(1) On a ak = k σ (Sk − Sk−1 ) (en posant S0 = 0).
n
X n
X
ak = k σ (Sk − Sk−1 )
k=1 k=1
n−1
X
σ
= n Sn + (k σ − (k + 1)σ )Sk .
k=1

P
n−1
Sn → a, on utilise l’égalité nσ = − (k σ −(k +1)σ )+1 d’où, en reportant dans l’égalité
k=1
ci-dessus :
n
X n−1
X
ak = Sn + (k σ − (k + 1)σ )(Sk − Sn )
k=1 k=1
n
X n−1
X
−σ −σ −σ
n ak = n Sn + n (k σ − (k + 1)σ )(Sk − Sn ).
k=1 k=1

n−σ Sn → 0 donc ce terme ne nous gène pas.


ε
Ensuite soit ε > 0, ∃N ∈ N | ∀n > N, |Sk − a| 6 d’où, par l’inégalité triangulaire,
4
|Sk − Sn | 6 ε/2.
SPÉCIALE MP* : ORAL 2006 23

On obtient finalement
n−1
X n−1
X
−σ σ σ −σ
n (k − (k + 1) )(Sk − Sn ) 6 n [(k + 1)σ − k σ ]ε
k=N k=N
  σ 
N
6ε 1− 6 ε.
n
La suite (Sn ) est bornée (suite convergente) par M donc |Sk − Sn | 6 2M, on choisit
NP
−1 ε
alors n suffisamment grand (n > N ′ > N) pour que n−σ (k σ − (k + 1)σ )|Sk − Sn | 6
k=1 4
ε
(on a une somme finie) et pour que |n−σ Sn | 6 d’où, pour n > N ′ ,
4
n
X N
X −1 n
X
−σ −σ −σ
n ak 6 n ak + n ak ε
k=1 k=1 k=N

P
n
i.e. lim n−σ ak = 0.
n→+∞ k=1
(2) On partage le plan en 4 parties délimitées par les droites y = ±x. On note J1 = {z =
π
x + iy | − x < y 6 x}, J2 , J3 , J4 se déduisant de J1 par rotations successives d’angle .
P 2
On note Si = |zi | et on suppose que S1 est la valeur maximale (le problème étant
i∈Ji
invariant par rotations, cette supposition est licite). On a alors

X
|zi | = S1 + S2 + S3 + S4 6 4S1
i∈J
Xq
64 x2i + yi2
i∈J1
√ X
64 2 xi
i∈J1

car yi2 6 x2i par définition de J1

√ X X
64 2 zi 6 6 zi
i∈J1 i∈J1
 
P P
car xi = Re zi et la partie réelle est inférieure au module.
i∈J1 i∈J1

Solution 2.5.1 (D’après R. Portalez).


(1) On peut utiliser la propriété suivante : si K passe par un extremum sur S(0, 1) alors le
K(x)
gradient de K est orthogonal à la sphère. On va plutôt considérer H(x) = défini
kxk4
sur Rn \ {0}. Si K passe par un extremum en x alors H passe par un extremum en µx
pour tout µ > 0 et on a équivalence donc grad H(x) = 0.
On utilise alors les propriétés suivantes :
grad(Ax|x) = 2Ax, grad(f g)(x) = f (x) grad g(x) + g(x) grad f (x),
24 SPÉCIALE MP* : ORAL 2006

1 K(x)
d’où grad H(x) = 4
[2Ax(A−1 x|x) + 2A−1 x(Ax|x)] − 4 x = 0 ce qui donne bien
kxk kxk6
le résultat avec λ = 2K(x).
(2) On compose par A la relation (Ax|x)A−1 x + (A−1 x|x)Ax = λx, on obtient alors
(A−1 x|x)A2 x − λAx + (Ax|x)x = 0 soit αA2 x + βAx + γx = 0 avec α > 0 et γ > 0.
αA2 +βA+γIn n’est pas inversible et se factorise sur C sous la forme (A−µIn )(A−νIn ).
Or si µ et ν sont complexes (conjuguées) non réelles alors A − µIn et A − νIn sont in-
versibles ce qui n’est pas le cas. µ et ν sont donc réelles.
• Si µ 6= ν alors, par le lemme des noyaux, x ∈ Eµ (A) + Eν (A).
P
• Si µ = ν on a (A − µIn )2 x = 0. Si x = xω est la décomposition de x dans la
ω∈Sp(A)
somme directe des sous-espaces propres de A alors
 
X
(A − µIn )2 x = (A − µIn )2  xω 
ω∈Sp(A)
X
= (A − µIn )2 xω = 0
| {z }
ω∈Sp(A) ∈Eω (A)

et comme la somme est directe, on en déduit que (A − µIn )2 xω = 0. Si ω 6= µ alors


(A − µIn )xω = (ω − µ)2 xω = 0 ⇒ xω = 0 donc x = xµ ∈ Eµ (A).
Dans les deux cas, on a x ∈ Eµ (A) + Eν (A).
(3) Soient λ1 6 λ2 6 . . . 6 λn les valeurs propres de A rangées dans l’ordre croissant.
Si x est un extremum de K sur la sphère alors x ∈ Eλi (A) + Eλj (A) et, en écrivant
x = xi ei + xj ej ,
1 1
K(x) = (λi x + λj x|x)( x + x|x)
λi λj
 
4 4 2 2 λj λi
= xi + xj + xi xj +
λi λj
(λi − λj )2
= (x2i + x2j )2 +x2i x2j >1
| {z } λi λj
=1

On en déduit tout d’abord que min K(x) > 1 et que ce minimum est atteint pour chaque
vecteur propre de A.
1 λ1 λn λi λj
f (x) = x + est croissante sur [1, +∞[ donc f ( λλji ) 6 f ( λλn1 ) soit + > +
x λn λ1 λj λi
donc
 
4 4 2 2 λn λ1
K(x) 6 xi + xj + xi xj +
λ1 λn
(λn − λ1 )2
6 1 + xy
λ1 λn
1 (λn − λ1 )2
avec x = x2i , y = x2j = 1 − x. x(1 − x) 6 donc K(x) 6 1 + et cette valeur
4 4λ1 λn
1
est atteinte pour x = √ (e1 + en ).
2

Solution 2.5.2 (F. Germain) Note : 12


(1) On démontre que
• si a < 1 alors un → a,
SPÉCIALE MP* : ORAL 2006 25

• si a > 1 alors u2n+1 → b < a et u2n → c > a avec bc = 1 (ou le contraire).


u1 > 0 donc un > 0 pour n > 1.
a(1 + a2 )2x
f ′ (x) = − 6 0, f est décroissante, f ◦ f est croissante donc les suites (u2n )
(1 + x2 )2
et (u2n+1 ) sont monotones. Or 0 6 un 6 a(1 + a2 ) donc elles sont aussi bornées.
Première conclusion : vn = u2n → b et wn = u2n+1 → c.
a(1 + a2 ) a(1 + a2 )
On a wn = et vn+1 = donc, en passant à la limite on obtient les
1 + vn2 1 + wn2
relations
a(1 + a2 ) a(1 + a2 )
c= et b =
1 + b2 1 + c2
2 2
donc (1 + b )c = b(1 + c ) ce qui équivaut à (c − b)(1 − bc) = 0 donc b = c ou bc = 1.
Là, les choses se compliquent.
• L’équation f (x) = x se traduit par (x − a)(x2 + ax + a2 + 1) = 0 donc a est le seul
point fixe.
• L’équation f ◦f (x) = x se traduit par (x−a)(x2 +ax+a2 +1)(x2 −a(1+a2 )x+1) = 0
(la factorisation se fait car, si on se place sur C alors f (z) = z ⇒ f ◦ f (z) = z donc
les racines (même complexes) de l’équation f (x) = x sont aussi racines de l’équation
f ◦ f (x) = x).
On est donc amené à chercher les points fixes de f ◦ f (qui vont donner les valeurs
possibles des limites de (vn ) et (wn )).
• Si 0 < a 6 1, l’équation f ◦ f (x) = x n’a qu’une seule solution réelle a (triple si
a = 1).
• Si a > 1 alors cette équation a trois racines b < a < c (avec bc = 1).
2a2
– |f ′ (a)| = > 1 donc le point a est répulsif, la suite (un ) ne peut tendre
1 + a2
vers a (sauf si u0 = a auquel cas elle est stationnaire).
– Il ne reste donc que l’autre possibilité i.e. vn → b et wn → c (ou le contraire).
(2)(⇒) Soit G un supplémentaire (commun) de Ker f (et de Im f ) E = Ker f ⊕ G et on sait
que f|G est bijective (appliquer le théorème du rang). Soit πF le projecteur sur F
parallèlement à G et h = g −1 ◦ πF .
Soit x = xF + xG ,
h ◦ f (x) = h(f (xG )) = g −1 ◦ f (xG ) = xG ,
f ◦ h(x) = f (g −1(xF )) = xF .
Conclusion : h ◦ f (x) + f ◦ h(x) = xG + xF = x.
(⇐) f 2 = 0 ⇒ Im f ⊂ Ker f . Si x ∈ Ker f alors f ◦h(x) = x donc x ∈ Im f d’où l’égalité.

Solution 3.1.1 (R. Portalez) Note : ?


µ
(1) L’équation homogène est une équation d’Euler et ses solutions sont données par λx + .
x
On cherche ensuite une solution D.S.E... (passionnant).
(2) A est symétrique réelle donc elle est diagonalisable dans une b.o.n.
Soit Pn (x) le polynôme caractéristique de A, en développant par rapport à la dernière
ligne, on trouve Pn (x) = (1 − x)Pn−1 (x) − (1 − x)n−2 soit
Pn (x) Pn−1 (x) 1
n
= n−1
− .
(1 − x) (1 − x) (1 − x)2
Pn (x) n−1
Ceci donne une suite arithmétique et P1 (x) = 1 − x d’où n
= 1− ce
(1 − x) (1 − x)2
qui s’écrit encore Pn (x) = (1 − x)n−2 [(1 − x)2 − (n − 1)].
26 SPÉCIALE MP* : ORAL 2006

E1 (A) est un sous-espace vectoriel de dimension n − 2, E1+√n−1 et E1−√n−1 sont des


sous-espaces propres de dimension 1.
(3) M est une matrice orthogonale ssi
p2 + q 2 + r 2 = 1 et σ2 = pq + qr + rp = 0
i.e. : (p + q + r)2 = p2 + q 2 + r 2 + 2σ2 = 1.
On a donc p + q + r = ±1 et σ2 = 0.
Maintenant, M est la matrice d’une rotation ssi det M = 1 i.e. p3 + q 3 + r 3 − 3pqr = −1
ce qui est encore équivalent (en factorisant) à (p + q + r)(p2 + q 2 + r 2 − σ2 ) = −1 i.e. :
p + q + r = −1.
On aura donc l’équivalence suivante : M est la matrice d’une rotation ssi p, q, r sont
les racines réelles de x3 + x2 + k = 0 (on connaı̂t les fonctions symétriques élémentaires
4
de ces racines) et la condition pour que ces racines soient réelles est 0 > k > − (on
27
utilise les formules de Cardan ou l’étude de f (x) = x3 + x2 + k).
P
N
(4) Soit fN = (−1)n e−an x . On complète cette somme par des termes nuls ce qui permet
n=0
d’appliquer le théorème des séries alternées et donc de majorer fN . Ce même
P théorème
permet aussi d’affirmer que la série (avec tous les termes cette fois-ci) (−1)n e−an x
vérifie les bonnes hypothèses :
(−1)n e−an x est une suite de signes alternés, décroissante vers 0 pour x > 0.
On applique alors le théorème de convergence dominée :
• |fN (x)| 6 e−a0 x = ϕ(x) et ϕ ∈ L1 (]0, +∞[),
C.S. P
+∞
• fn −−−−→ (−1)n e−an x
]0,+∞[ n=0
donc
Z +∞ N
X (−1)n
lim fN (x) dx = lim
N →+∞ 0 N →+∞
n=0
an
Z +∞  
= lim fN (x) dx
0 N →+∞
Z +∞
!
+∞ X
= (−1)n e−an x dx
0 n=0

ce qui fournit la formule demandée.

Solution 3.1.2 (Th. Pradeau) Note : 8


Examinateur : celui de l’équipe d’oral 4 (le même qu’Antoine). Barbu, a passé tout son temps à
pianoter sur son pc, mais est quand même resté calme et sympa vu comment je m’embrouillais
pitoyablement sur des trucs simples.
P
n
(1) a) Alors on peut faire comme moi et partir en live avec Pn (X) = X i − 1 et retrouver
i=1
une suite géométrique pour X 6= 1, ou alors ouvrir grand les yeux et voir que
Pn (0) = −1 et Pn (1) > 0 pour n > 1. Comme Pn est continue, d’après le TVI on a
l’existence. De plus Pn′ (X) = nX n−1 + ... + 2X + 1 > 1 > 0 pour X > 0, donc Pn
est strictement croissante, et bijective sur R+ d’où l’unicité.
b) Pas trouvé, attention aux vannes rapides genre Pn (X) −→ P (X) et P ( 21 ) = 0 donc
an −→ 12 !!! Il n’a pas trop aimé !
c) Pas fait.
SPÉCIALE MP* : ORAL 2006 27

(2) Là j’ai compris que c’était fini, surtout que je ne suis pas arrivé à faire cet exo non plus
!!
On montre que toute famille finie (fαi )i∈[|1,n|] est libre (les αi étant différents) en
procédant par récurrence. Pour n = 1, c’est immédiat, on suppose que c’est vrai au
P
n
rang n − 1. Supposons alors qu’on ait λi fαi = 0, et considérons k | αk = sup αi ,
i=1 i∈[|1,n|]
αk t
P ai t
P (ai −αk )t
on a alors λk t e =− λi t e , soit λk t = − λi t e . On fait tendre t vers +∞,
i6=k i6=k
le membre de droite tend vers 0 car αi − αk < 0 pour tout i 6= k ; on obtient ainsi
λk t −→ 0 soit λk = 0. On applique ensuite la propriété de récurrence et on obtient que
t−→∞
tous les λi sont nuls, d’où la propriété au rang n, et la liberté de la famille (fα )α∈R .
Une autre façon intéressante, avec fα (t) = eαt (oui je galérais beaucoup...) : on considère
l’opérateur dérivation D ∈ L(C ∞ (R)), et on a alors D(fα ) = fα′ = αfα . On a ainsi fαi
qui est un vecteur propre de D associé à la valeur propre αi . Comme toute famille de
vecteurs propres associés à des valeurs propres différentes est libre, on a bien le résultat.

Solution 3.1.3 (G. Bouyt) Note : ?


Examinateur : M. Guillaumé, très sympathique ; donne des indications quand il le faut.
Pn−1 k
(1) On constate tout d’abord que comme 1 − xn = (1 − x) k=0 x , In est bien définie.
Ensuite, on voit que
xn (1 − x) xn
∀n ∈ N, ∀x ∈ [0; 1], n
= n−1
6 xn 6 1.
1−x 1 + x + ... + x
Donc on peut appliquer le théorème de convergence dominée, et conclure que (In )n∈N
converge vers 0.
On transforme ensuite In , grâce au changement de variable t = xn :
Z
1 1 t1/n (1 − t1/n )
In = dt.
n 0 1−t
1
Puis on procède à une intégration par parties, en intégrant 1−t :
Z 1
 1/n

1/n 1 1 ln(1 − t) 1/n
In = − ln(1 − t) t (1 − t) 0
+ 2 t (1 − 2t1/n ) dt.
n 0 t
Le crochet est nul ; de plus,
ln(1 − t) 1/n ln(1 − t)
∀n ∈ N, ∀t ∈ [0; 1], t (1 − 2t1/n ) 6 − .
t t
Et t 7→ − ln(1−t)
t
est intégrable sur [0; 1], donc en utilisant le théorème de convergence
dominée, on obtient que :
Z 1
1 ln(1 − t)
In ∼ − 2 dt.
n 0 t
P
Ceci assure que In converge. Le calcul de la somme est une autre histoire...
(2) On note F et G respectivement l’image et le noyau de u. Comme u est un projecteur,
u2 = u ; on remarque ainsi que u est vecteur propre de Φ associé à la valeur propre 2.
Introduisant v, projecteur sur G parallèlement à F , on voit que v est vecteur propre de
Φ associé à la valeur propre 0. On cherche alors un polynôme annulateur pour Φ en
s’aidant de ces premières remarques. On calcule, pour f ∈ L(E):
Φ(Φ − 2id)(f ) = 2uf u − Φ(f ).
28 SPÉCIALE MP* : ORAL 2006

On réapplique Φ :
Φ2 (Φ − 2id)(f ) = 2uf u − Φ(f ) = Φ(Φ − 2id)(f ).
Ainsi, Φ annule le polynôme scindé sur R à racines simples X(X − 1)(X − 2), donc Φ
est diagonalisable.
On peut préciser les vecteurs propres associés à la valeur propre 1 : ce sont les endo-
morphismes qui envoient F sur G et G sur F .

Solution 3.1.4 (G. Alary) Note : 8


Examinateur : muet et ne montre absolument pas si on est ou pas sur la bonne voie : j’ai dit
une grosse ânerie et j’ai continué l’exercice en l’utilisant, il ne m’a à aucun moment reprise, il
m’a montré l’erreur juste avant de me dire au revoir !
(1) C’est le lemme des noyaux.
(2) C’est du cours...
(3) M est une matrice orthogonale ssi
p2 + q 2 + r 2 = 1 et σ2 = pq + qr + rp = 0
i.e. : (p + q + r)2 = p2 + q 2 + r 2 + 2σ2 = 1.
On a donc p + q + r = ±1 et σ2 = 0.
Maintenant, M est la matrice d’une rotation ssi det M = 1 i.e. p3 + q 3 + r 3 − 3pqr = −1
ce qui est encore équivalent (en factorisant) à (p + q + r)(p2 + q 2 + r 2 − σ2 ) = −1 i.e. :
p + q + r = −1.
On aura donc l’équivalence suivante : M est la matrice d’une rotation ssi p, q, r sont
les racines réelles de x3 + x2 + k = 0 (on connaı̂t les fonctions symétriques élémentaires
4
de ces racines) et la condition pour que ces racines soient réelles est 0 > k > − (on
27
3 2
utilise les formules de Cardan ou l’étude de f (x) = x + x + k).

Solution 3.1.5 (G. Polchi) Note : ?


n
Y
(1) a) Sens direct : P est de la forme : P (X) = (X − λp ) où λp ∈ R.
p=1
n
Y
En prenant z = a + ib, P (z) = (a − λp + ib). Ainsi
p=1
n
Y Y 2
|P (z)|2 = [(a − λp )2 + b2 > n|b|2 = |b|deg P .
p=1 p=1

Réciproque : il suffit de montrer que les racines de P sont réelles ce qui est immédiat
en prenant l’inégalité appliquée aux racines : P (z) = 0 donc Im(λ) = 0.
b) U n’est pas nécessairement diagonalisable.  Contre
 exemple : on prend l’endo-
0 1
morphisme de matrice associée : A = qui n’est pas diagonalisable, et
0 0
 
1/m 1
Am = est diagonalisable et Am → A.
0 0
U est trigonalisable. Pour cela il suffit de montrer que son polynôme caractéristique
est scindé sur R et d’après 1 il suffit donc de montrer l’inégalité. Or comme Um est
diagonalisable on a l’inégalité vraie pour Pm (polynôme caractéristique de Um ) et
en passant à la limite sur m, comme Pm → P on a ce qu’on veut.
(2)
SPÉCIALE MP* : ORAL 2006 29

Solution 3.1.6 (M. André) Note : 16


Examinateur : sympa mais détaché, demande de justifier quelques trucs au début puis laisse
faire. J’ai appris la veille que je n’étais pas admissible aux ENS aussi je n’étais pas très motivé
et pas très réveillé. Fubini m’a fatigué mais on s’en sort...
Z X Z X
(1) f est positive et f (x, y) dy admet une limite en +∞ notée ϕ(x), ϕ(x) dx admet
1 1
aussi une limite. On fait de même en inversant les rôles de x et y. Fubini s’applique et
donne le résultat.
(2) a) ⇒ On sait que kxk2 = kp(x)k2 + kx − p(x)k2 (Pythagore) donc kp(x)k 6 kxk.
⇐ Là, j’ai trouvé l’astuce sordide et il a juste dit “Oui”, vu la note il a dû croire
que je l’avais déjà fait...
Il suffit de montrer que Ker p ⊥ Im p. Soit y ∈ Ker p et x ∈ Im p, t > 0 on a
p(ty + x) = x donc kxk2 6 kty + xk2 et, en développant, 2t(y|x) + t2 kyk2 > 0. On
simplifie alors par t > 0 et on prend la limite quand t → 0 d’où (y|x) > 0. En
faisant le même raisonnement avec t < 0 on obtient (y|x) 6 0 soit (y|x) = 0.
b) Tout projecteur orthogonal vérifie kpk 6 1 (on a en fait égalité si on écarte le
projecteur nul), donc K est borné.
Par continuité de ◦ et de k.k, K est fermé (kpn k → kpk et p2n → p2 ).
Conclusion : K est un fermé borné en dimension finie, c’est un compact.
(3) Pour terminer la planche (les 2 premiers exos ont été rapidement expédié) une courbe
en polaires !

Solution 3.1.7 (P. Adroguer) Note : 9


Examinateur : rien à dire mais son exo était beaucoup trop “fougéreux” (?). Je suis passé sans
préparation pendant 45 minutes.
(1) P R un
un dx
(2) Si un converge alors un → 0 donc 1+u n
∼ u n , ln(1+u n ) ∼ u n , 0 1+x2
= Arctan un ∼
un P
ce qui assure la convergence de toutes les séries.
Si un diverge.

Solution 3.1.8 (J. Assoun) Note : 15


Jusqu’à la dernière question et malgré un examinateur assez froid et distant, qui ne dit rien,
c’est assez simple, d’où l’intérêt de connaı̂tre les quadriques (ce qui n’était pas mon cas).
˙ = 20
(1) En remplaçant y, on trouve 28x ˙ et, en simplifiant par 4, on obtient 7x ≡ 5[9] soit
x ≡ 2[9]. On est ramené à chercher les éléments de Z/36Z qui valent 2 dans Z/9Z.
Conclusion : l’ensemble des couples solutions est donné par
˙ (11,
{(2̇, 35), ˙ 8̇), (20,
˙ 17),
˙ (29,
˙ 26)}.
˙

(2) f ′′ > 0 donc f ′ ր et a une limite l dans R.


Si l > 0 alors f n’est pas majorée ce qui est contraire à l’hypothèse donc l 6 0.
Si l < 0 alors f n’est pas minorée, idem. On a donc l = 0.
f ′ est donc négative, f est décroissante et minorée donc convergente et admet une limite
m > 0. f > m donc f ′′ > α2 m et, par le même argument que pour f ′ , on prouve que
m = 0.
30 SPÉCIALE MP* : ORAL 2006

(3) On écrit les équivalences suivantes :

M(X, Y, Z) ∈ Σ ⇔ ∃P (x, y, z) ∈ C | M = r(P )(r rotation d’axe OZ)


⇔ ∃(x, y, z) ∈ C | Z = z, X 2 + Y 2 = x2 + y 2
⇔ x + z = 1, (x − 1)2 + y 2 + z 2 = 1, Z = z, X 2 + Y 2 = x2 + y 2
⇔ X 2 + Y 2 + Z 2 − 2(1 − Z) = 0

la dernière équivalence s’obtient en prenant z = Z, x = 1 −√Z. On obtient alors


X 2 + Y 2 + (Z + 1)2 = 3 sphère de centre (0, 0, −1) et de rayon 3.
(4) Cf. cours.

Solution 3.1.9 (J.C. Martin) Note : ?


Examinateur : ?
(1) a) Montrons par l’absurde que ∆ n’existe pas : en effet, si ∆ existe alors ∆2 (1) = 0
donc ∆(1) ∈ Ker ∆ or D(∆(1)) = ∆3 (1) = 0 donc ∆(1) = k (constante). On en
déduit que ∆(k) = 0.
Si k 6= 0 alors ∆(k) = k∆(1) = 0 ce qui est impossible.
Si k = 0 alors ∆(1) = 0. D(x) = 1 donc ∆D(x) = 0 soit D∆(x) = 0 i.e. ∆(x) est
constant donc ∆2 (x) = D(x) = 0 ce qui est encore impossible.
b) Il suffit de définir ∆ par ∆(a cos x + b sin x) = a cos(x + π4 ) + b sin(x + π4 ).
sin xt
(2) a) Soit F (t, x) = définie pour (t, x) ∈ [0, +∞[×]0, +∞[.
x(1 + x2 )
• F (., x) est continue sur [0, +∞[, F (t, .) est continue sur ]0, +∞[,
∂F | cos(xt)| 1
• Pour tout t, (t, x) 6 2
6 qui est intégrable.
∂t 1+x 1 + x2
En application du théorème de continuité sous le signe intégral, f est continue sur
[0, +∞[.
∂F cos xt ∂F
(t, x) = 2
est continue par rapport à x et par rapport à t et (t, x) 6
∂t 1+x ∂t
1
.
1 + x2 Z +∞
1 ′ cos xt
On en déduit que f est de classe C sur [0, +∞[ et que f (x) = dx.
0 1 + x2
b) On ne peut pas appliquer directement le théorème de dérivation sous le signe intégral
1
à f ′ , on va faire une intégration par parties : on intègre cos xt et on dérive 1+x 2 d’où

Z
′ 1 +∞ 2x sin xt dx 1
f (t) = 2 2
= g(t).
t 0 (1 + x ) t
Par le théorème de dérivation sous le signe intégral, on peut dériver g donc f ′ est
dérivable sur ]0, +∞[ et on a
Z +∞
′′ 1 ′ 1 x/t cos xt − 1/t2 sin xt
f (t) = g (t) − 2 g(t) = 2x dx
t t 0 (1 + x2 )2
Z X
x/t cos xt − 1/t2 sin xt
= lim 2x dx
X→+∞ 0 (1 + x2 )2
Z X
−x sin xt
= lim dx
X→+∞ 0 1 + x2
SPÉCIALE MP* : ORAL 2006 31

2x
en faisant une autre intégration par partie (on intègre et on dérive
(1 + x2 )2
x/t cos xt − 1/t2 sin xt). On peut donc écrire
Z +∞
′′ x sin xt
f (t) = − dx
0 1 + x2
Z +∞
′′ sin xt π
et donc f (t) − f (t) = dx = (on a fait le changement de variable
0 x 2
R +∞
u = xt et on a utilisé le célèbre résultat 0 sinu u du = π2 ).
π π
Comme f (0) = 0 et f ′ (0) = , on en déduit que f (t) = (1 − e−t ).
2 2
π
c) f est impaire donc, si t < 0 alors f (t) = −f (−t) = − (1 − et ).
2
Conclusion : on peut donner l’expression générale de f sous la forme f (t) =
π e−|t|/2 sh t/2.

Solution 3.1.10 (B. Dobre) Note : ?


Examinateur :
(1) a) ϕ est une application linéaire en dimension finie donc est continue.
b) On note P = a0 + a1 X + · · · + an X n .
• Norme 1 : |ϕ(P )| 6 |a0 | + |a1 |.|a| + · · · + |an |.|a|n 6 max(1, |a|n )kP k1 donc
kϕk 6 max(1, |a|n ).
Si |a| 6 1 on a égalité avec P = 1.
Si |a| > 1, on prend P = X n pour avoir égalité.
Conclusion : ici on a prouvé que kP k = max(1, |a|n ).
• Norme infinie : |ϕ(P )| 6 (1 + |a| + · · · + |a|n )kP k∞ , maximum atteint pour
– P = 1 + X + · · · + X n si a > 0,
– P = 1 − X + · · · + (−1)n X n si a 6 0.
Conclusion : kϕk = 1 + |a| + · · · + |a|n .
(2) No comment
(3) Soit on fait un beau dessin avec des couleurs soit on fait un raisonnement (les deux sont
acceptables ici à condition que ce soit bien fait).
On utilise l’équivalence (S désigne la surface recherchée)
( (
x2 + y 2 − 4z = 9 Z =z
M(X, Y, Z) ∈ S ⇔ ∃(x, y, z) | vérifiant .
x + y − 2z = 0 X 2 + Y 2 = x2 + y 2
S est contenue dans le paraboloı̈de elliptique de révolution d’équation X 2 + Y 2 − 4Z = 9
(évident à priori) mais il y a des limitations sur Z que l’on va chercher.
Si on élimine z entre les équations de la courbe, on obtient la projection orthogonale
sur le plan xOy(: c’est un cercle d’équation (x − 1)2 + (y − 1)2 = 11 que l’on peut
√ r
x = 1 + 11 cos θ x+y 11
paramétrer par √ d’où z = =1+ sin(θ + π4 ) et au final
y = 1 + 11 sin θ 2 2
q q
z ∈ [1 − 11 2
, 1 + 11
2
].
Conclusion : S est la portion de paraboloı̈de elliptiqueqde révolution quiqadmet l’équation
11 11
X 2 + Y 2 − 4Z = 9 délimitée par les plans Z1 = 1 − 2
et Z2 = 1 + 2
q
(si Z est compris entre ces deux valeurs extrêmes, on pose z = Z = 1 + 112
sin(θ + π4 ),
√ √
x = 1 + 11 cos θ, y = 1 + 11 sin θ donc pour M(X, Y, Z) vérifiant X 2 + Y 2 − 4Z = 9,
on a bien m(x, y, z) qui appartient à la courbe).
32 SPÉCIALE MP* : ORAL 2006

(4) Comme X 4 +5X 2 +4 = (X 2 +1)(X 2 +4) alors les valeurs propres (qui sont nécessairement
racines du polynôme annulateur) sont contenues dans l’ensemble {±i, ±2i}. Comme A
est réelle, son polynôme caractéristique est réel. Si i est racine d’ordre k alors −i est
racine d’ordre k aussi, de même, si 2i est racine d’ordre l alors −2i est racine d’ordre l.
La trace d’une matrice est égale à la somme de ses valeurs propres (réelles et complexes)
comptées avec leur ordre de multiplicité donc
Tr(A) = ki + k(−i) + l2i + l(−2i) = 0.

Solution 4.1.1 (Th. Pradeau) Note : 12


M. Vallaeys, avait l’air agacé pendant la colle, pourtant pour une fois je ne disais pas de vannes.
Mis à part, il est très sympa : le gars d’avant n’avait rien fait du tout et il a pris 5 minutes à
la fin pour discuter de ses concours, des CCP et des écoles auquel il est admissible, et lui a dit
de ne pas se décourager. C’est assez rare à Centrale pour être remarqué !
Pn
(1) a) A ∈ On (R), donc ses colonnes sont de norme 1, soit ∀j ∈ [|1, n|], a2ij = 1, on a
i=1
P
ainsi a2ij = n. On utilise ensuite Cauchy-Schwarz :
i,j
v ! !
u
X u X X √ √
|aij |×1 6 t a2ij 1 = n×n2 = n n.
i,j i,j i,j
 Pn 
i=1 a1j
.. P
b) Av =   donc |(Av|v)| = aij . Or |(Av|v)| 6 |Av| |v|, et il faut se
Pn . i,j
anj
i=1
souvenir qu’un endomorphisme orthogonal est par définition une isométrie (il m’en
P
a voulu !) donc |Av| = |v| . Ainsi, aij = |(Av|v)| 6 |v|2 = n.
i,j
 
|a11 |
c) Si on a égalité dans a. i.e. dans Cauchy-Schwarz, les vecteurs considérés  ... 
|ann |
 
1
et  ...  sont proportionnels, donc ∀(i, j), |aij | = λ. Les colonnes étant de norme 1,
1  
1
on doit avoir λ = n1 , donc A = n1  ou . Dans ce cas, on a l’égalité dans b.
−1
P
aij = n1 ×n2 ssi il n’y a que des 1 ou que des −1 dans la matrice (immédiat :
i,j
sinon la somme des coefficients ne fait pas n2 en valeur absolue). Ceci n’est bien sur
pas possible car A est orthogonale (donc de déterminant non nul).
d) On suppose A = AT = A−1 . Comme A est réelle symétrique,elle est diagona-
  1
λ1 0 λ1
0
.  .  T
lisable : A = P  ..  P et A = P 
T −1 ..  P (les λi sont
1
0 λn 0 λn
non nuls car A est inversible). On en déduit λi = λ1i , soit λi = ±1 pour tout
i. Dans une certaine base orthonormée, la matrice de l’endomorphisme s’écrit
SPÉCIALE MP* : ORAL 2006 33
 
1 0
 .. 
 . 
 
 1 
 , donc on trouve bien que c’est une symétrie orthogo-
 −1 
 . 
 .. 
0 −1
nale (mais apparemment il ne voulait pas que je le fasse aussi rigoureusement...).
(2) On veut montrer que ce groupe est monogène et fini. Il est clair qu’il est fini, et qu’il

Z 2
a 16 éléments. Soit (a, b) ∈ 4Z , en réfléchissant un peu on voit que l’ordre de (a, b)

Z 2 Z
dans 4Z est le ppcm de l’ordre de a et de celui de b dans 4Z . Comme 4a = 4b = 0,
l’ordre de (a, b) est d’au plus 4, donc ce couple permet après itérations d’obtenir au

Z 2

Z 2
plus 4 couples différents de 4Z . Or, 4Z a 16 éléments, donc aucun couple ne peut
l’engendrer, i.e. il n’est pas monogène et a fortiori pas cyclique.
(3) J’y ai réfléchi juste 2 minutes et j’avais pas vu que c’était des formes linéaires, il n’a pas
trop apprécié et a décidé de s’arrêter là.

Solution 4.1.2 (J. Charrel) Note : ?


Examinateur agréable qui aide beaucoup et qui ne fait pas trop attention a la précision.
(1)

Solution 4.1.3 (G. Alary) Note : 16


Examinateur : pressé de finir sa journée (je passais en dernier) mais sympa, a été étonné que
je bloque sur la première question et m’a questionné sur la logique élémentaire (si je demande
”Est-ce que toutes les personnes ayant 3 bras conduisent une voiture ?” on doit répondre “Oui”
car l’hypothèse de départ est fausse).
(1) On a Ker ϕf = {P ∈ K[X] | P (f ) = 0} = 6 {0} qui est un idéal donc Ker ϕf = πf K[X]
d’où l’existence du polynôme minimal.
(2) On a πf (f ) = 0 et πf n’admet pas 0 comme racine (sinon, f étant inversible, on pourrait
π
prendre Xf ) donc πf (f ) = 0 = am f m + · · · + a0 Id avec a0 6= 0 d’où
   
1 m−1 1 m−1
f◦ (am f + · · · + a1 Id) = (am f + · · · + a1 Id) ◦ f = Id
a0 a0
donc l’inverse de f est un polynôme en f .
(3) En dimension finie, tout endomorphisme admet un polynôme annulateur. On prend le
contre-exemple en dimension infinie. Soit D l’opérateur dérivation sur l’ensemble des
polynômes, s’il existait un polynôme annulateur de D, on aurait
am D m + · · · + a0 Id = 0 avec (am , . . . , a0 ) 6= (0, . . . , 0).
En appliquant cette relation au monôme X m on arriverait au fait que la famille
(1, X, . . . , X m ) est liée, ce qui est faux.
(4) Le polynôme minimal divise le polynôme caractéristique (qui est un polynôme annula-
teur), on peut donc dire que si M admet X 6 + X 4 + X 2 + 1 comme polynôme minimal
alors n > 6.

Solution 4.1.4 (N. Charon) Note : ?


Examinateur : Mr Franchini. Environ une quarantaine d’années, franchement sympathique, il
34 SPÉCIALE MP* : ORAL 2006

aide par contre un peu trop vite (mais c’est peut-être aussi parce que je n’avais pas fait grand-
chose pendant les 20 mn de préparation). Son exo est assez intéressant et demande d’avoir bien
assimilé les notions de formes linéaires et de base anté-duale.
(1) Soit S le polynôme minimal de M de degré p et P ∈ C[X]. On fait la division euclidienne
de P par S :
P = SQ + R, avec R ∈ Cp−1 [X]. Alors P (M) = R(M) et C(M) = Cp−1 [M]. De plus, la
famille (In , M, M 2 , . . . , M p−1 ) est nécessairement libre dans C(M) car sinon on aurait
un polynôme annulateur de degré < au polynôme minimal. Donc dim C(M) = p (on a
vu ce résultat dans les compléments d’algèbre à la fin de l’année).
Y
On suppose ensuite M diagonalisable. Dans ce cas, S = (X − λ). Soit A ∈ IM ,
λ∈Sp(M )
2
A = P (M) avec P ∈ Cp−1 [X]. Alors P − 1 est un polynôme annulateur de M donc on
peut écrire :
P 2 − 1 = SQ ⇒ P 2 = SQ + 1.
Là, je n’ai pas tellement su comment continuer alors qu’il suffit de remarquer simple-
ment que ∀λi ∈ Sp(M), P 2 (λi ) = 1 ce qui suggère de décomposer P dans la base des
Pp
polynômes interpolateurs de Lagrange aux points λi : P (X) = ai Li (X). On obtient
i=1
alors ai = P (λi) = ±1 ce qui donne 2p possibilités pour P .
Réciproquement, on vérifie que tous ces polynômes sont bien dans IM d’où Card Im =
2p = 2Card(Sp(M )) .
(2) Ici, bien que les hypothèses soient différentes, la démarche est assez semblable. On
suppose M nilpotente et on appelle k le plus petit entier tel que M k = 0. Ainsi X k
est le polynôme minimal de M. Ensuite, on prend comme précédemment A = P (M)
dans IM . P 2 = QX k + 1. On a alors P 2 (0) = 1 et ∀i ∈ [[1, k − 1]], P (i) (0) = 0. En
X k−1
décomposant P dans la base anté-duale : (1, X, . . . , (k−1)! ), on a P 2 = 1 soit P = ±1.
Ainsi Card IM = 2 = 2Card IM car Sp(M) = {0}.
(3) Je n’ai pas eu le temps finir cette question pendant la colle. Au début, vu les deux
questions d’avant, je m’étais jeté sur une décomposition de Dunford alors que ça ne sert
à rien. En fait, on veut montrer que pour toute matrice M, Card IM = 2Card Sp(M ) et pour
cela on reprend une démarche analogue. M étant trigonalisable sur C, on peut écrire le
p
Y
polynôme minimal sous la forme T (X) = (X − λi )ωi avec ωi ∈ N (là, c’est peut-être
j=1
une arnaque ou une vanne mais il me semble qu’en utilisant la réduite de Jordan, ça a
l’air de passer). On prend A = P (M) dans IM , deg P 6 deg T − 1. Alors P 2 = QT + 1.
On a ainsi ∀i ∈ [[1, p]], P 2(λi ) = 1 et ∀i ∈ [[1, p]], ∀j ∈ [[0, ωi − 1]], P (j) (λi ) = 0. On
considère alors la famille (ϕij ) de formes linéaires de Rk [X], (k = ω1 , . . . , ωp − 1) définie
par : ϕij (P ) = P (j) (ωi ). Il suffirait alors de faire intervenir la base anté-duale de cette
famille mais encore faut-il que ce soit bien une base de Rk [X]. Je pense qu’ici on peut
p
Y
montrer qu’elle est libre en utilisant des polynômes adéquats du genre (X − λi )αi
i=1
mais j’ai un peu la flemme de le faire. Enfin toujours est-il que en utilisant la base
anté-duale, on trouve 2p possibilités pour P , comme attendu.

Solution 4.1.5 (G. Polchi) Note : ?


SPÉCIALE MP* : ORAL 2006 35

Solution 4.1.6 (P. Adroguer) Note : 12


Examinatrice : Mme Joly, m’a posé tout le temps des questions débiles, a fait la maligne en
parlant de classe d’équivalence, un régal !
(1) On remarque que A + AT = J − In où J = (1) (ne pas écrire Id pour la matrice identité
sinon crise de nerf).
Soit X ∈ Ker A ∩ H, on remarque que H = Ker J. On a donc AT X = −X, AX = 0
donc AT X = 0 et (AX)T X = X T AT X = 0 soit −X T X = −kXk2 = 0 par conséquent
X = 0.
Conclusion : Ker A ∩ H = {0}.
(2) Immédiat avec la formule du rang : dim Ker A 6 1 car Ker A et H sont en somme
directe.  
0 1
(3) n = 2 alors A = ou sa transposée donc Rg(A) = 1.
0 0
 
0 0 ... 0 1
 .. 
1 . 0
. . . .
Si n > 3 alors A =  
 .. . . . . 0 ..  est de rang n.
 . . . . .. 
1 . . .
0 1 ... 1 0
Conclusion : n = 2.
Questions annexes :
(1) 0 est d’ordre n − 1 et n d’ordre 1 (question : c’est quoi l’ordre de multiplicité d’une
vap ?).
(2) J’ai calculé det A en développant par rapport à la première ligne mais ça ne lui a pas
plu.
(3) J 2 = nJ, le polynôme caractéristique de J est (−X n−1 )(n−X) et son polynôme minimal
vaut X(X − n).  
0 −1
(4) Il suffit de prendre A = .
1 0

Solution 4.1.7 (J. Assoun) Note : 17


Examinateur : Mr Franchini, très sympathique, un vrai dialogue s’installe.
Premier exo bidon, il a vu que je connaissais des résultats généraux sur le commutant d’une
matrice à valeurs propres simples et du coup : “Bon, eh bien puisque vous m’en avez trop dit,
vous m’avez donné l’idée d’une deuxième question”. Avis aux amateurs : si vous voulez un bel
exo d’algèbre, bien au dessus de ce qui est “normal” à Centrale, provoquez l’examinateur !
(1) a) Par Cayley-Hamilton, on sait que I = 6 {0} et on en déduit que I est engendré par
ΠA , polynôme minimal de A. Or PA = (X + 1)(X − 2)2 et ΠA |PA , de plus −1 et 2
sont racines de ΠA donc ΠA = PA ou ΠA = (X + 1)(X − 2). Le calcul montre que
ΠA = (X + 1)(X − 2). On a ainsi I = (X + 1)(X − 2)R[X].
On a donc A2 = A+2I3 et, par une récurrence élémentaire, ∀n ∈ N, An = an I3 +bn A
avec an+1 = 2bn et bn+1 = an + bn . Comme la famille (I3 , A) est libre, on en déduit
que dim R[A] = 2.
b) (an ) vérifie alors la relation de récurrence an+2 − an+1 − 2an = 0 qui se résout
immédiatement sous la forme an = α(−1)n + β2n . Finalement, on trouve
2 1 2 2
an = (−1)n + 2n et bn = an+1 = − (−1)n + 2n .
3 3 3 3
c) Soit B ∈ CR (A). A est symétrique réelle donc diagonalisable et comme A et B
commutent, E−1 (A) et E2 (A) sont stables par B. On a en fait équivalence et si on
36 SPÉCIALE MP* : ORAL 2006

considère les endomorphismes a et b de R3 de matrices A et B alors, dans une base


diagonalisante pour a, on aura A′ = Diag(−1, 2, 2) et B ′ = Diag(α, B2 ) où B2 est
une matrice carrée d’ordre 2.
Conclusion : dim CR (A) = 1 + 4 = 5.
d) Si B 2 = A alors B ∈ CR (A) donc, avec les notations de la question précédente,
α2 = −1 ce qui est impossible.
Conclusion : A n’a pas de “racine carrée” dans R.
(2) Si PA est scindé à racines simples et si B ∈ CC (A), B stabilise tous les sous-espaces
propres de A qui sont de dimension 1 donc A et B sont simultanément diagonalisables
dans une base (ei ). On en déduit que dim CC (A) = n (B est déterminé de manière
unique par les B(ei )).
On utilise alors la densité de l’ensemble D des matrices diagonalisables à racines simples
dans Mn (C) : soit (Ap ) une suite de matrices de D qui tend vers A et ϕp : X ∈
Mn (C) 7→ Ap X − XAp . On sait, d’après ce qui précède que dim Ker ϕp = n, en outre,
ϕp converge simplement vers ϕ définie par ϕ(X) = AX − XA. Or, dans un e.v.n.
de dimension finie, la convergence simple d’une suite d’endomorphismes entraı̂ne la
convergence de cette suite d’endomorphismes (ϕp (Ei ) converge pour (Ei ) base et on
prend la norme infinie dans cette base).
On utilise enfin la propriété suivante : dim Ker ϕ > lim dim Ker ϕp . En effet, si on
p→+∞
note M et Mp les matrices de ϕ et ϕp dans une base alors, si r = Rg ϕ, on sait qu’il
existe un mineur d’ordre r extrait de M non nul, par continuité du déterminant, ce
même mineur sera non nul dans Mp (à partir d’un certain rang) donc Rg Mp > r soit
dim CC (A) = Rg(ϕ) > Rg(ϕp ) = n.

Solution 4.1.8 (B. Dobre) Note : ?


 
0 α
(1) a) A = 0 alors X = , α ∈ R donne une infinité de solutions.
0 0
 
0 1
A= ne donne pas de solution.
1 0
   
1 0 ε√
1 0
A= donne 4 solutions X = .
0 2 ε2 2
 
0 1 0
b) Si A = 0 0 1 alors A3 = 0 donc X 6 = 0 et (par Cayley-Hamilton) X 3 = 0.

0 0 0
On a alors X 2 (e2 ) = e1 et X 2 (e3 ) = e2 or X 3 (e3 ) = X(e2 ) = 0 ce qui est impossible
(X(e2 ) =0 ⇒ X 2 (e2 ) = 0.
2 0 0
Si A = 1 2 0 alors comme X et A commutent, on sait que X et A sont
0 1 3
co-trigonalisables et là, on s’amuse...
c) Si χA est scindé à racines simples alors toute matrice qui commute avec A est
diagonalisable dans la même base que A. X est donc diagonalisable et X 2 = A
impose la positivité des valeurs propres de A.
Si A a toutes ses valeurs propres positives alors, comme au troisième exemple du a,
on aura 2n solutions.
Si A a au moins une valeur propre strictement négative alors on n’a aucune solution.
d) On peut trigonaliser simultanément A et X, on pourra trouver des solutions en
résolvant un système triangulaire (ARNAQUE).
e) Pas de solution.
SPÉCIALE MP* : ORAL 2006 37
 
λ1 In1 0 0
f) On se place dans une base diagonalisante, A =  0 ..
. 0 . XA = AX ⇒
  0 0 λp Inp
X1 0 0
X = 0 0  (même découpage que pour A et on résout Xk2 = λk Ink .
0 0 Xp
g) X  est une matrice
 de symétrie diagonalisable, les solutions sont données par X =
Ip
P 0  P −1 (à l’ordre des vecteurs près).
0 −Im
(2) On montre par récurrence que la famille des dérivées est libre :
Si λ1 cos x + · · · + λn nxn−1 cos xn = 0 alors en x = 0 on obtient λ1 = 0. On simplifie
par x et on recommence. Par une récurrence sabotée (mais finie) on arrive à λ1 = λ2 =
. . . = λn = 0.
Supposons maintenant que λ1 sin x + λ2 sin x2 + · · · + λn sin xn = 0 alors, en dérivant,
on obtient λ1 cos x + · · · + λn nxn−1 cos xn = 0 donc les λi sont tous nuls, la famille est
bien libre.

Solution 4.1.9 (F. Germain) Note : ?


(1)

Solution 4.2.1 (Th. Pradeau) Note : 10


Mme Péry, très antipathique pendant l’oral. Elle a du voir que je n’étais pas trop stressé pour
ce concours et elle s’énervait dès que je proposais une piste qui ne marchait (peut-être) pas en
répondant avec mépris que ça servait à rien. J’ai retrouvé son charmant état d’esprit dans ma
note...
(1) Déjà, an est décroissante et minorée donc elle converge, et en plus elle est bornée par
a0 . P
a) Si t = 1, comme un (1) = 0, on a convergence de un .
PN P n P n+1
N N P
N
Pour t 6= 1, on a un (t) = an t − an t = a0 − aN tN − (an − an−1 )tn . Or
n=0 n=0 n=0 n=1
P
N
aN tN −→ 0 et |an − an−1 | tn 6 2a0 tn , donc (an − an−1 )tn converge, car |t| < 1.
N −→∞
P n=1
Donc un converge.  n
n an n
b) u′n (t) = an tn−1 (n − (n + 1)t) donc |un |∞ = un n+1 = n+1 n+1
. Or
 n   
n 1
ln = −n ln 1 + = −1 + o(1),
n+1 n
P
d’où |un |∞ ∼ 1e n+1
an
, avec que des termes positifs, donc un converge normalement
P an
ssi n+1
converge.
P

c) Notons l > 0 la limite de an et RN (t) = an tn (1 − t) le reste de la série. Comme
n=N +1
P
∞ P
∞ P

an décroı̂t on a l tn (1−t) 6 RN (t) 6 aN tn (1−t). Or tn (1−t) = tN +1
n=N +1 n=N +1 n=N +1
donc l×tN +1 6 RN (t) 6 aN ×tN +1 , or |t| 6 1 donc on a 0 6 l 6 ||RN ||∞ 6 aN . Donc
par encadrement ||RN ||∞ −→ l et on a convergence uniforme ssi l = 0.
d) Sur tout compact [0, b], b 6= 1 on a un (t) 6 a0 bn (1 − t) 6 a0 bn , donc on a convergence
normale et on peut appliquer le théorème de continuité. Montrons maintenant qu’il
38 SPÉCIALE MP* : ORAL 2006


X ∞
X
n P

n’y a pas continuité en 1 : on a pour t 6= 1, l t (1 − t) 6 un (t) 6 a0 tn (1 − t)
n=0
|n=0 {z } |n=0 {z }
=1 =1
P

donc, puisque l 6= 0, 0 < l 6 un (t) = u(t). Or u(1) = 0, donc on ne peut pas
n=0
avoir continuité en 1.
(2) Je ne comprends toujours pas cette question : on utilise le théorème de double limite,
puisqu’on a convergence uniforme :
P
N P∞ P∞
lim lim fn (t) = lim lim fn (t) soit encore lim f (t) = αn ce qui assure la
t−→∞N −→∞n=0 N −→∞t−→∞n=0 t−→∞
P n=0
convergence de αn et la limite de f pour t −→ ∞. Y-a-t’il une vanne ?

Solution 4.2.2 (J. Charrel) Note : ?


(1) a) On trouve une relation de récurrence:
∆n+1 = ∆n + un+1vn+1 ∆n−1 .
De là, on déduit que ∆n est croissante et on cherche à la majorer:
∆n 6 (1 + un+1vn+1 )∆n−1
∆n 6 wn ∆0 .
Puis on majore wn :
n
X n
X
ln wn = ln (1 + uk vk ) 6 uk vk .
k=1 k=1
Donc ∆n converge.
b) Avec la relation de récurrence:
n−1
X
(∆n ) CV ⇒ ∆k+1 − ∆k CV
k=0
Pn−1 Pn−1
∆k+1 − ∆k = k=0 uk+1 vk+1 ∆k−1
k=0 Pn−1
> k=0 uk+1 vk+1
X
⇒ uk vk converge.
 √ √
(1 − √tn ) et n si t 6 n
(2) On pose fn (t) = √
0 si t > n
√ 2 √
n ln (1− √t )+t n −t
n( √ − t +o(1/n))+t n 2 /2+o(1)
or fn (t) = e n = e n 2n = e−t .
2
⇒ fn (t) −→ e−t /2 ⇒ puis domination ...???

Solution 4.2.3 (G. Alary) Note : 16


Examinateur : très sympa, pour la deuxième question, je lui ai fait le cas q = 1 qui est trivial
mais je n’avais par pensé à la récurrence, je cherchais à décomposer la fraction ; il a apprécié
que je fasse le “petit cas”. J’ai fini l’exercice en 2 minutes mais il n’en a pas donné d’autre, il
a préféré parler des valeurs approchées de π.
(1) On a
1 1 1
− = qun,q où un,q =
n(n + 1)(. . .)(n + q − 1) (n + 1)(. . .)(n + q) n(n + 1)(. . .)(n + q)
on a une série aux différences ce qui donne tout de suite le résultat.
SPÉCIALE MP* : ORAL 2006 39

(2) On fait une récurrence sur q :


P
+∞ 1 P 1
+∞
• q = 1 : il suffit de prouver que 1 + 2
= 2
. On fait la différence des
n=1 n (n + 1) n=1 n
deux sommes d’où
+∞
X +∞
X
1 1 1
2
− 2 = =1
n=1
n n (n + 1) n=1 n(n + 1)
d’après le (1) avec q = 1.
• On suppose la propriété vraie à l’ordre q, là il suffit de montrer que
+∞
X +∞
X
1 1 1
q! = + (q + 1)! .
n=1
n2 (n + 1)(. . .)(n + q) (q + 1)2 n=1
n2 (n + 1)(. . .)(n + q + 1)

On fait à nouveau la différence des deux sommes en utilisant le résultat suivant


q! (q + 1)! q!
2
− 2 = .
n (n + 1)(. . .)(n + q) n (n + 1)(. . .)(n + q + 1) n(n + 1)(. . .)(n + q + 1)
d’où
+∞
X X +∞
1 1 1
q! 2
− (q + 1)! 2
= q!
n=1
n (n + 1)(. . .)(n + q) n=1
n (n + 1)(. . .)(n + q + 1) (q + 1)(q + 1)!
(toujours en utilisant le résultat du (1). On simplifie par q! et on obtient le résultat.
P
+∞ 1
(3) On majore le reste de q! 2
, soit
n=1 n (n + 1)(. . .)(n + q)
+∞
X 1
rN =
n=N +1
n2 (n + 1)(. . .)(n + q)
+∞
X 1
6
n=N +1
nq+1
Z +∞
1 1
6 dt = .
N tq+2 (q + 1)N q+1
q!
On cherche à avoir < 10−100 , à N fixé, cherchons la plus petite valeur de
(q + 1)N q+1
q! uq+1 (q + 1)2
la suite uq = : = rapport < 1 tant que q + 1 6 N donc la
(q + 1)N q+1 uq (q + 2)N
plus petite valeur de la suite uq est obtenue pour q = N − 1. On a alors
 q q p √
q! 1 2π
uq = q+2
∼ 2πq q+2 = 3/2 q+1 .
(q + 1) e q e q e
Avec Maple, on obtient u223 = 0, 4.10−100 et u222 = 1, 07.10−100 .
P 1
222
On peut prendre alors q = 222, N = 223, calculer 2
avec une précision de 0, 2.10−100
n=1 n
P
223 1
par défaut, 222! 2
avec une précision de 0, 2.10−100 par défaut.
n=1 n (n + 1)(. . .)(n + 222)
Si on appelle A la valeur exacte et à la valeur approchée alors A − 0, 4.10−100 6 à 6 A
π2
et = A + R223 avec R223 < 1, 07.10−100 d’où
6
π2
à 6 6 à + 1, 47.10−100
6
40 SPÉCIALE MP* : ORAL 2006

π2
et on peut alors prendre comme valeur approchée de à 10−100 près la valeur à +
6
0, 7.10−100 (par exemple).

Solution 4.2.4 (G. Polchi) Note : ?


(1) C’est le lemme de Lebesgue.
Z π/2 Z π/2
cos x cos(x/2)
On a sin(2nx) dx = sin(nx) dx. On voudrait appliquer le
0 sin x 0 2 sin(x/2)
1. mais ici la fonction n’est pas C 1 d’où l’étude de θ :
1 1
θ(x) = − = o(x) → 0
2 tan(x/2) x
donc θ est continue sur [0, π/2]. On calcule θ′ et on fait de même pour montrer qu’elle
est continue. Ainsi θ est C 1 et on applique 1. Au final la limite cherchée est 0.
(2)

Solution 4.2.5 (P. Adroguer) Note : 12


Examinateur : Mr Fairbank, aimable. Les 2 exo ont tenu 10 minutes, la colle aussi.
2 2 2
(1) a) Si x = 0 la série diverge, si x 6= 0 alors e−x n < e−x n donc la série converge.
2 2
b) On utilise le théorème de comparaison série-intégrale : soit ϕ(t) = e−x t alors
Z +∞ Z +∞
−x2 t2 2 2
x e dt > xf (x) − x > x e−x t dt.
0 1

π
On e déduit que lim xf (x) = .
x→0 2
c) On a convergence normale de la série définissant f pour x > a > 0 donc on peut
appliquer le théorème de double limite, lim f (x) = 0.
x→+∞
(2) On fait un DL de ln(1 − 1/n) à l’ordre 4 :
   
2 1 1 1 1
un = cos −n π + 2 + 3 +O
n 2n 3n n4
  
π π 1
= cos −nπ − − +O
2 3n n2
  
π 1
= (−1)n+1 sin +O
3n n2
= vn + wn
 
n +1 π 1 P P
où vn = (−1) et wn = O 2
. vn converge par le T.S.A. et wn converge
3n n
par comparaison
P à une série de Riemann.
Conclusion : un converge.

Solution 4.2.6 (J. Assoun) Note : 10


Examinatrice : (Mme Joly ?) assez sympathique, attentive et intéressée. Je n’ai pas calculé S
avec Maple mais à la main et elle a accepté (ce qu’elle n’a, vu ma note, pourtant pas apprécié,
belle hypocrisie).
SPÉCIALE MP* : ORAL 2006 41
 
1 1
(1) a) Le terme général vaut =O ce qui assure la
(4n + 1)(4n + 2)(4n + 3)(4n + 4) n4
convergence.
P
+∞ x4n+4
Pour le calcul, on introduit f (x) = pour |x| <
n=0 (4n + 1)(4n + 2)(4n + 3)(4n + 4)
1. f est de classe C ∞ (série entière) et f (0) = f ′ (0) = f ′′ (0) = f ′′′ (0) donc, en
utilisant la formule de Taylor avec le reste intégral, on obtient
Z
1 x (x − t)3
f (x) = dt.
6 0 1 − t4
Z
1 1 (1 − t)3 (1 − t)3
Or f est continue en 1 (S converge) donc S = f (1) = dt. =
6 0 1 − t4 1 − t4
(1 − t)2
et la décomposition de cette fraction rationnelle donne
(1 + t)(1 + t2 )
(1 − t)2 2 t+1
2
= −
(1 + t)(1 + t ) 1 + t 1 + t2
Z 1
(1 − t)2 3 π ln 2 π
d’où 2
dt = ln 2 − soit S = − (résultat confirmé par
0 (1 + t)(1 + t ) 2 4 4 24
Maple).
PS : le résultat donné par Julien est faux, ceci justifie peut-être la note...
1
b) Soit g(x) = alors g est positive, décroissante et
(4x + 1)(4x + 2)(4x + 3)(4x + 4)
intégrable sur R+ , le théorème de comparaison série-intégrale s’applique et on a
Z +∞ Z +∞
g(t) dt 6 rn 6 g(t) dt.
n+1 n
1
Or g(t) ∼ , par intégration des relations de comparaison, on a donc rn ∼
256t4
1
.
768n3
x
(2) a) un (x) ∼ 2 d’où la convergence simple sur R+ .
n
b
Si x ∈ [a, b] ⊂ R+ alors 0 6 un (x) 6 2 ce qui assure la convergence normale
n + a2
sur tout segment.
b) On applique le théorème de comparaison série-intégrale à fx qui est continue et
décroissante sur R+ d’où
Z +∞ +∞
X Z +∞
fx (t) dt 6 un (x) 6 fx (t) dt .
1 0
| {z } n=0 | {z }
=π/2−Arctan 1/x =π/2

P
+∞ π
c) On en déduit alors que lim un (x) = .
P x→+∞ n=0 2
Si un (x) convergeait uniformément sur R+ alors, en appliquant le théorème
d’interversion d’une somme et d’une limite, on aurait
+∞
X +∞
X
lim un (x) = lim un (x) = 0
x→+∞ x→+∞
n=0 n=0 | {z }
=0

ce qui est contradictoire.


P
Conclusion : la série un (x) ne converge par uniformément sur R+ .
42 SPÉCIALE MP* : ORAL 2006

Solution 4.2.7 (B. Dobre) Note : ?


Examinateur : ?
(1) Trop simple.
1 1
(2) On prend les inverses : = + un−1 d’où, en sommant
un+1 un
1 1
= + u0 + u1 + · · · + un−1 = a + Sn−1 .
un+1 u1 | {z }
=Sn−1

1 1 1
Or = = a + Sn−1 soit Sn+1 − Sn = .
Sn+1 − Sn un+1 a + Sn−1
La suite (un ) est décroissante (un+1 6 un ) et positive donc un → l > 0. En passant à
la limite dans la relation de récurrence, on obtient l = 0 donc a + Sn−1 → +∞ d’où
Sn → +∞ et Sn+1 − Sn → 0.
2 Sn+1 + Sn
Ensuite Sn+1 − Sn2 = → 2, en utilisant le théorème de Césaro, on a
a + Sn−1
n−1
Sn2 S2 1 X 2
= 0 + [S − Sk2 ]
n n n k=0 k+1
→2
√ 1
donc Sn2 ∼ 2n et par conséquent Sn ∼ 2n et finalement un ∼ √ .
2n

Solution 4.2.8 (F. Germain) Note : ?


Z 1
xn 1
(1) n−1 2
n
6 x donc un 6 xn dx = → 0.
(1 + x + · · · + x ) 0 n+1
(2) Là ça devient un peu plus technique :
Z 1 n
x (1 − x)2
un = dx et on développe en série la fonction intégrée :
0 (1 − xn )2
+∞
xn (1 − x)2 X n
n 2
= x (1 − x)2 (p + 1) (xn )p
(1 − x ) p=0
+∞
X
= fp (x) (on a translaté l’indice)
p=1

où fp (x) = p(1 − x)2 xnp est une fonction positive. On utilise alors le théorème de
Lebesgue
Z 1 d’intégration terme à terme d’une série de fonctions :
1 2 1 2p
fp (x) dx = − + = terme général
0 np + 1 np + 2 np + 3 (np + 1)(np + 2)(np + 3)
d’une série convergente donc on peut intégrer terme à terme d’où
+∞
X 2p
un =
p=1
(np + 1)(np + 2)(np + 3)
2
= vn
n3
SPÉCIALE MP* : ORAL 2006 43

P
+∞ p 1
où vn = vn,p avec vn,p = . Or vn,p 6 2 donc la conver-
p=1 (p + 1/n)(p + 2/n)(p + 3/n) p
gence de cette série est normale par rapport à la variable 1/n, on a donc
+∞
X
lim vn = lim vn,p
n→+∞ n→+∞
p=1
+∞
X 1 π2
= =
p=1
p2 6
π2
et en conclusion : un ∼ .
3n3

Vous aimerez peut-être aussi